[obm-l] Re: [obm-l] equação do 2 grau métodos de sol

2013-08-05 Por tôpico Esdras Muniz
x² - 3x + 5 = 0
x² - (3x/2) + (3/2)² = -5 + (3/2)²
(x - 3/2)² = (3/2)² - 5



Em 5 de agosto de 2013 12:06, Hermann ilhadepaqu...@bol.com.br escreveu:

 **
 Dei aula para um peruano que não usava báskara, mas não tive tempo na
 época.

 Alguém aqui saberia me explicar outros métodos de se obter a solução da
 equação (sem báskara, sem S e P)

  ax^2+bx+c=0

 abraços

 Hermann

 --
 Esta mensagem foi verificada pelo sistema de antivírus e
 acredita-se estar livre de perigo.




-- 
Esdras Muniz Mota
Graduando em Matemática Bacharelado
Universidade Federal do Ceará

Se algum dia ele recuou, foi para dar um grande salto

-- 
Esta mensagem foi verificada pelo sistema de antivírus e
 acredita-se estar livre de perigo.



Re: [obm-l] Enigma de Martin Gardner

2013-08-19 Por tôpico Esdras Muniz
679



Em 17 de agosto de 2013 22:30, Mauricio de Araujo 
mauricio.de.ara...@gmail.com escreveu:

 Persistência de um número é o número de passos necessários para reduzi-lo
 a um único dígito multiplicando todos os seus algarismos para obter um
 segundo número, depois multiplicando todos os dígitos deste número para se
 obter um terceiro número, e assim por diante, até que um número de um
 dígito é obtido. Por exemplo, 77 tem uma persistência de quatro, porque
 requer quatro etapas para reduzi-lo a um dígito: 77-49-36-18-8. O menor
 número de persistência 1 é 10, o menor de persistência 2 é 25, o menor de
 persistência 3 é 39, e o menor de persistência 4 é 77. Qual é o menor
 número de persistência cinco?

 --
 Abraços

 oɾnɐɹɐ ǝp oıɔıɹnɐɯ
 *momentos excepcionais pedem ações excepcionais.*
 *Os cemitérios estão cheios de pessoas insubstituíveis em seus ofícios.*

 --
 Esta mensagem foi verificada pelo sistema de antivírus e
 acredita-se estar livre de perigo.




-- 
Esdras Muniz Mota
Graduando em Matemática Bacharelado
Universidade Federal do Ceará

Se algum dia ele recuou, foi para dar um grande salto

-- 
Esta mensagem foi verificada pelo sistema de antiv�rus e
 acredita-se estar livre de perigo.

#include stdio.h
#include stdlib.h
#include math.h

int proddig(int n)
{
int k,l,p=1;
k=n;
while(k1)
{
   if(k%10!=0) p=p*(k%10);
   k=k-(k%10);
   k=k/10;
}
return(p);
}
int main()
{int i,k,cont=1;
for(k=1;k1000;k++)
{
i=k;
cont=0;
 while(i9)
 {  
 i=proddig(i);
cont++;
 }
 
 printf(%d__%d\t,k,cont);
 if(cont==5)printf(Aqui!);
 printf(\n);
}
   return 0;
}


Re: [obm-l] O Paradoxo da Flecha - Off Topic

2013-08-27 Por tôpico Esdras Muniz
Imagino que todas as soluções considerm o tempo contínuo, mas me parece que
quem criou essa situação considerava o tempo discreto.


Em 27 de agosto de 2013 10:09, luiz silva luizfelipec...@yahoo.com.brescreveu:

 Pessoal,

 Mais uma tentativa de responder para a lista :

 O surgimento das Teorias da Relatividade (Geral e Restrita) trouxe consigo
 algumas consequências filosóficas bastante curiosas que foram
 identificadas naquela época por Eddington, Weyl, Einstein, Minkowski, etc e
 que até hoje é muito discutida (creio que ainda está em aberto).

 Basicamente, as TRs nos levaram a concluir que não existe fluxo de tempo
 (apesar de termos a seta do tempo) e que, com isso, vivemos em um Block
 Universe (BU). Nesta visão, passado, presente e futuro co-existem um
 universo 4D (objeto físico), não existe um agora objetivo. Neste
 universo, não existe fluxo de tempo, processos; nada acontece, nada flui,
 as coisas apenas são. O que existem são eventos congelados, fixos neste
 espaço-tempo.

 Esta visão aparentemente é sustentada pela maioria dos físicos
 relativistas. Uns falam que a propria TRR já nos levaria a um BU (vide O
 Paradoxo de Andrômeda), outros falam que a união de espaço e tempo feita
 por Minkowski nos leva a esta conclusão. Claro que existe muita gente que
 não concorda, principalmente os quânticos (apesar de que, alguns
 relativistas defendem que a MQ deveria se adequar a TR e ser desenvolvida
 considerando-se o BU).

 Ou seja, o verdadeiro problema por trás do paradoxo da flecha é o nosso
 conhecimento das entidades básicas do nosso universo (espaço e/-tempo). Se
 realmente os relativistas estão corretos, e vivemos em um BU, então
 realmente a flecha está parada, e o paradoxo realmente existe; se vivemos
 em um universo do tipo Newtoniano, em construção, então concordo com o
 que foi falado aqui, e o que temos é um falso paradoxo.

 O problema é que se este realmente for um verdadeiro paradoxo (ou seja, se
 vivemos em um BU), então este problema inocente é muito mais sério do que
 parece, pois vai envolver nada mais nada menos que nossa percepção
 consciente.

 Em anexo, estou enviando dois arquivos. Um artigo escrito por mim, e
 outro que é parte do livro de um físico chamado Vasselin Petkov;
 basicamente o que eu falo e ele também, era defendido (não sei se da mesma
 forma) por Weyl, Eddington, Minkowski, Godell e outros.

 Além disso, seguem alguns links com documentários sobre esta questão e
 outros artigos sobre a questão do BU.

 http://www.youtube.com/watch?v=oNSEoYU8VSs (este da BBC é realmente MUITO
 BOM - é a primeira parte)
 http://www.youtube.com/watch?v=j-u1aaltiq4 (este é um do Brain Greene -
 trecho especifico onde fala da questão do BU)

 Artigos :
 http://www.fqxi.org/data/essay-contest-files/Wharton_Wharton_Essay.pdf
 http://philsci-archive.pitt.edu/2408/1/Petkov-BlockUniverse.pdf

 Espero que se divirtam :-)
 Abs
 Felipe

   --
  *De:* luiz silva luizfelipec...@yahoo.com.br
 *Para:* obm-l@mat.puc-rio.br obm-l@mat.puc-rio.br
 *Enviadas:* Segunda-feira, 26 de Agosto de 2013 23:35

 *Assunto:* Re: [obm-l] O Paradoxo da Flecha - Off Topic

 Ola Pessoal,

 Infelizmente minhas respostas não estão entrando na lista; assim, tomei a
 liberdade de enviar a minha resposta em pvt, para os que responderam.

 Abs
 Felipe

   --
  *De:* Ralph Teixeira ralp...@gmail.com
 *Para:* obm-l@mat.puc-rio.br
 *Enviadas:* Segunda-feira, 26 de Agosto de 2013 21:47
 *Assunto:* Re: [obm-l] O Paradoxo da Flecha - Off Topic

 2013/8/26 Bernardo Freitas Paulo da Costa bernardo...@gmail.com:
  Já temos um problema aqui. Ao considerar que num instante, a flecha
  está parada. Nada disso, ela TEM velocidade, mas observar o movimento
  só faz sentido AO LONGO do tempo, não numa fotografia, e é por isso
  que você não vê a flecha se mexer nesta dita fotografia.

 zero!

 Oi, Paulo Cesar, eu ia responder... mas, po, o Bernardo expressou
 exatamente o que eu penso do assunto nesse paragrafo ai em cima, de
 maneira mais clara do que eu seria capaz de dizer. :) :)

 Abraco,
 Ralph

 --
 Esta mensagem foi verificada pelo sistema de antivírus e
 acredita-se estar livre de perigo.


 =
 Instruções para entrar na lista, sair da lista e usar a lista em
 http://www.mat.puc-rio.br/~obmlistas/obm-l.html
 =



 --
 Esta mensagem foi verificada pelo sistema de antivírus e
 acredita-se estar livre de perigo.



 --
 Esta mensagem foi verificada pelo sistema de antivírus e
 acredita-se estar livre de perigo.




-- 
Esdras Muniz Mota
Graduando em Matemática Bacharelado
Universidade Federal do Ceará

Se algum dia ele recuou, foi para dar um grande salto

-- 
Esta mensagem foi verificada pelo sistema de antivírus e
 acredita-se estar livre de perigo.



Re: [obm-l] CN 2009

2013-09-05 Por tôpico Esdras Muniz
O produto das areas é (1*2)*(2*3)*...*(99*100) = (99!)*(100!).
Agora basta ver que a maior potência de 5 que divide 99! é 5^24
e a maior potência de 5 que divide 100! é 5^24.
Assim, a maior potencia de 5 que divide 99!100!  46 coincide com a
 maior potencia de 10 que divide este número.
Logo, termina com 46 zeros.


Em 4 de setembro de 2013 22:50, Hermann ilhadepaqu...@bol.com.br escreveu:

 **
 A explicação de como fiz: como temos fatores 2 em abundância, nem me
 preocupei e só calculei quantos fatores 5 tínhamos e contei 44 fatores.
 Abraços
 Hermann

 - Original Message -
 *From:* Hermann ilhadepaqu...@bol.com.br
 *To:* obm-l@mat.puc-rio.br
 *Sent:* Wednesday, September 04, 2013 10:08 PM
 *Subject:* Re: [obm-l] CN 2009

 Refiz e deu 44 zeros, esqueci-me que o númeo era repetido, acho que agora
 acertei.

 - Original Message -
 *From:* Paulo Barclay Ribeiro paulobarc...@yahoo.com.br
 *To:* obm-l@mat.puc-rio.br
 *Sent:* Wednesday, September 04, 2013 7:46 PM
 *Subject:* [obm-l] CN 2009

  Prezados, Boa noite.

 Peço uma orientação para resolver a seguinte questão do Colegio Naval de
 2009:

 Sobre o lado maior de um retangulode base 1 e altura 2 constroi-se um
 retangulo de base 2 e altura 3;sobre o maior lado desse último constroi-se
 um retangulo de base 3 e altura 4; e assim sucessivamente,até se construir
 o retangulo de base 99 e altura 100.Com quanto zeros termina o produto das
 areas de cada um desses retangulos?
 Desde já agradeço

 Um abraço
 paulo Barclay

 --
 Esta mensagem foi verificada pelo sistema de antivírus e
 acredita-se estar livre de perigo.


 --
 Esta mensagem foi verificada pelo sistema de antivírus e
 acredita-se estar livre de perigo.


 --
 Esta mensagem foi verificada pelo sistema de antivírus e
 acredita-se estar livre de perigo.




-- 
Esdras Muniz Mota
Graduando em Matemática Bacharelado
Universidade Federal do Ceará

Se algum dia ele recuou, foi para dar um grande salto

-- 
Esta mensagem foi verificada pelo sistema de antivírus e
 acredita-se estar livre de perigo.



Re: [obm-l] CN 2009

2013-09-05 Por tôpico Esdras Muniz
Obrigado.


Em 5 de setembro de 2013 12:13, Hermann ilhadepaqu...@bol.com.br escreveu:

 **
 Meu amigo, vou corrigir sua excelente solução:
  Agora basta ver que a maior potência de 5 que divide 99! é 5^22  ()
 ok
 Abraços
 Hermann

 - Original Message -
 *From:* Esdras Muniz esdrasmunizm...@gmail.com
 *To:* obm-l@mat.puc-rio.br
 *Sent:* Thursday, September 05, 2013 11:20 AM
 *Subject:* Re: [obm-l] CN 2009

 O produto das areas é (1*2)*(2*3)*...*(99*100) = (99!)*(100!).
 Agora basta ver que a maior potência de 5 que divide 99! é 5^24
 e a maior potência de 5 que divide 100! é 5^24.
 Assim, a maior potencia de 5 que divide 99!100!  46 coincide com a
  maior potencia de 10 que divide este número.
 Logo, termina com 46 zeros.


 Em 4 de setembro de 2013 22:50, Hermann ilhadepaqu...@bol.com.brescreveu:

 **
 A explicação de como fiz: como temos fatores 2 em abundância, nem me
 preocupei e só calculei quantos fatores 5 tínhamos e contei 44 fatores.
 Abraços
 Hermann

  - Original Message -
 *From:* Hermann ilhadepaqu...@bol.com.br
 *To:* obm-l@mat.puc-rio.br
  *Sent:* Wednesday, September 04, 2013 10:08 PM
 *Subject:* Re: [obm-l] CN 2009

 Refiz e deu 44 zeros, esqueci-me que o númeo era repetido, acho que agora
 acertei.

 - Original Message -
 *From:* Paulo Barclay Ribeiro paulobarc...@yahoo.com.br
 *To:* obm-l@mat.puc-rio.br
 *Sent:* Wednesday, September 04, 2013 7:46 PM
 *Subject:* [obm-l] CN 2009

  Prezados, Boa noite.

 Peço uma orientação para resolver a seguinte questão do Colegio Naval de
 2009:

 Sobre o lado maior de um retangulode base 1 e altura 2 constroi-se um
 retangulo de base 2 e altura 3;sobre o maior lado desse último constroi-se
 um retangulo de base 3 e altura 4; e assim sucessivamente,até se construir
 o retangulo de base 99 e altura 100.Com quanto zeros termina o produto das
 areas de cada um desses retangulos?
 Desde já agradeço

 Um abraço
 paulo Barclay

 --
 Esta mensagem foi verificada pelo sistema de antivírus e
 acredita-se estar livre de perigo.


 --
 Esta mensagem foi verificada pelo sistema de antivírus e
 acredita-se estar livre de perigo.


 --
 Esta mensagem foi verificada pelo sistema de antivírus e
 acredita-se estar livre de perigo.




 --
 Esdras Muniz Mota
 Graduando em Matemática Bacharelado
 Universidade Federal do Ceará

 Se algum dia ele recuou, foi para dar um grande salto

 --
 Esta mensagem foi verificada pelo sistema de antivírus e
 acredita-se estar livre de perigo.


 --
 Esta mensagem foi verificada pelo sistema de antivírus e
 acredita-se estar livre de perigo.




-- 
Esdras Muniz Mota
Graduando em Matemática Bacharelado
Universidade Federal do Ceará

Se algum dia ele recuou, foi para dar um grande salto

-- 
Esta mensagem foi verificada pelo sistema de antivírus e
 acredita-se estar livre de perigo.



Re: [obm-l] Irracional entre dois racionais

2013-09-16 Por tôpico Esdras Muniz
( raiz(65)-raiz(63) )( raiz(65)+raiz(63) ) = 2
mas raiz( (65+63)/2)(raiz(65) + raiz(63))/2 por MA=MQ
==
raiz(65) - raiz(63)1/8=0,125
raiz(65) - raiz(63),14=
20,14(raiz(65) + raiz(63))0,14*(8+7)=2,1


Em 16 de setembro de 2013 08:33, marcone augusto araújo borges 
marconeborge...@hotmail.com escreveu:

 O número raiz(65) - raiz(63) está mais próximo de:

 a) 0,12   b) 0,13   c) 0,14   d) 0,15   e) 0,16

 --
 Esta mensagem foi verificada pelo sistema de antivírus e
 acredita-se estar livre de perigo.




-- 
Esdras Muniz Mota
Graduando em Matemática Bacharelado
Universidade Federal do Ceará

Se algum dia ele recuou, foi para dar um grande salto

-- 
Esta mensagem foi verificada pelo sistema de antivírus e
 acredita-se estar livre de perigo.



[obm-l] Re: [obm-l] Aritmética

2013-09-19 Por tôpico Esdras Muniz
Use o seguinte fato:
se a,b pertencem aos inteiros positivos, |a²-b²|=2*min{a,b}+1.
A²=y²+4x
B²=x²+y+2
4x=A²-y²=2y+1
y+2=B²-x²=2x+1
então
y=2x-1/2
y=2x-1
então
2x-1=y=2x-1/2  elevando ao quadrado fica:
4x²-4x+1=y²=4x²-2y+1/4  somando 4x:
(2x)²+1=A²=(2x+1)²-2x+1/4
isto nos dá um absurdo, pois o roximo quadrado depois de(2x)² é (2x+1)².


Em 17 de setembro de 2013 15:33, marcone augusto araújo borges 
marconeborge...@hotmail.com escreveu:

 Prove que não existem inteiros positivos x,y tais que x^2 + y + 2 e 4x +
 y^2
 são ambos quadrados perfeitos

 Eu peço uma dica para essa.


 --
 Esta mensagem foi verificada pelo sistema de antivírus e
 acredita-se estar livre de perigo.




-- 
Esdras Muniz Mota
Graduando em Matemática Bacharelado
Universidade Federal do Ceará

Se algum dia ele recuou, foi para dar um grande salto

-- 
Esta mensagem foi verificada pelo sistema de antivírus e
 acredita-se estar livre de perigo.



[obm-l] Re: [obm-l] RE: [obm-l] RE: [obm-l] Polinômios

2013-09-25 Por tôpico Esdras Muniz
Tome o polinomioQ(y)= P(1/x) fazendo y=1/x, temos:
Q(y)=a(n) + a(n-1)y + ...+ a(3)y^(n-3) + y^(n-2) + y^(n-1) + y^n
sendo r1, r2, ..., rn as raizes de Q(y) (com repetição). note que se R1,
R2,..., Rn são as raizes de P(x), Ri=1/ri  (note que an é diferente de
zero, então Q não possui raiz nula)
Então:
r1+r2+...+rn=-1;
(soma sobre ij)(ri*rj)=1;
então (r1)²+(r2)²+...+(rn)²=(r1+r2+...+rn)²-2*(soma sobre ij)(ri*rj)= -1 -
2*1=-3.
Então não podemos ter todas as raízes reais.


Em 25 de setembro de 2013 12:51, marcone augusto araújo borges 
marconeborge...@hotmail.com escreveu:

 As expressões entre parêntesis na penúltima linha não são ambas iguais a 1?
 E por que ´´para n par...´´?




 --
 From: joao_maldona...@hotmail.com
 To: obm-l@mat.puc-rio.br
 Subject: [obm-l] RE: [obm-l] Polinômios
 Date: Tue, 24 Sep 2013 23:00:14 -0300


 Sendo cp = 1/ap
 a1a2...an =  +-1/an
 a1a2...an(1/a1 + 1/a2+...+1/an) =   -+1/an
 a1a2...an(1/a1a2 + 1/a1a3 +... +1/an-1an) =   +-1/an

 Logo: (1/a1 + 1/a2+...+1/an) = -1
 (1/a1a2 + 1/a1a3 +... +1/an-1an) =1
 x=c1+c2+ ... +cn = -1
 y=c1c2+c1c3+...+cn-1cn = 1

 c1²+c2² +... +cn² = (c1+c2+ ... +cn)² -2(c1c2+c1c3+...+cn-1cn) = -1,
 absurdo, logo para n par temos  que pelo menos 2 raízes são complexas

 []'s
 João


 --
 From: marconeborge...@hotmail.com
 To: obm-l@mat.puc-rio.br
 Subject: [obm-l] Polinômios
 Date: Wed, 25 Sep 2013 01:00:58 +

 Prove que um polinômio do tipo a(n)x^n + a(n-1)x^(n-1) + ...+ a(3)x^3 +
 x^2 + x + 1 com
 coeficientes reais não pode ter todas as raízes reais.


 --
 Esta mensagem foi verificada pelo sistema de antivírus e
 acredita-se estar livre de perigo.

 --
 Esta mensagem foi verificada pelo sistema de antivírus e
 acredita-se estar livre de perigo.

 --
 Esta mensagem foi verificada pelo sistema de antivírus e
 acredita-se estar livre de perigo.




-- 
Esdras Muniz Mota
Graduando em Matemática Bacharelado
Universidade Federal do Ceará

-- 
Esta mensagem foi verificada pelo sistema de antivírus e
 acredita-se estar livre de perigo.



[obm-l] Re: [obm-l] RE: [obm-l] Re: [obm-l] RE: [obm-l] RE: [obm-l] Polinômios

2013-09-26 Por tôpico Esdras Muniz
Q(y)=a(n) + a(n-1)y + ...+ a(3)y^(n-3) + y^(n-2) + y^(n-1) + y^n
pelas equações de Girard, a soma das raizers é dada por menos o coeficiente
de y^(n-1), ou seja, -1.



Em 25 de setembro de 2013 21:14, marcone augusto araújo borges 
marconeborge...@hotmail.com escreveu:

 Por que r1+r2+...+rn = -1?

 --
 From: esdrasmunizm...@gmail.com
 Date: Wed, 25 Sep 2013 13:28:35 -0300
 Subject: [obm-l] Re: [obm-l] RE: [obm-l] RE: [obm-l] Polinômios
 To: obm-l@mat.puc-rio.br


 Tome o polinomioQ(y)= P(1/x) fazendo y=1/x, temos:
 Q(y)=a(n) + a(n-1)y + ...+ a(3)y^(n-3) + y^(n-2) + y^(n-1) + y^n
 sendo r1, r2, ..., rn as raizes de Q(y) (com repetição). note que se R1,
 R2,..., Rn são as raizes de P(x), Ri=1/ri  (note que an é diferente de
 zero, então Q não possui raiz nula)
 Então:
 r1+r2+...+rn=-1;
 (soma sobre ij)(ri*rj)=1;
 então (r1)²+(r2)²+...+(rn)²=(r1+r2+...+rn)²-2*(soma sobre ij)(ri*rj)= -1
 - 2*1=-3.
 Então não podemos ter todas as raízes reais.


 Em 25 de setembro de 2013 12:51, marcone augusto araújo borges 
 marconeborge...@hotmail.com escreveu:

 As expressões entre parêntesis na penúltima linha não são ambas iguais a 1?
 E por que ´´para n par...´´?




 --
 From: joao_maldona...@hotmail.com
 To: obm-l@mat.puc-rio.br
 Subject: [obm-l] RE: [obm-l] Polinômios
 Date: Tue, 24 Sep 2013 23:00:14 -0300


 Sendo cp = 1/ap
 a1a2...an =  +-1/an
 a1a2...an(1/a1 + 1/a2+...+1/an) =   -+1/an
 a1a2...an(1/a1a2 + 1/a1a3 +... +1/an-1an) =   +-1/an

 Logo: (1/a1 + 1/a2+...+1/an) = -1
 (1/a1a2 + 1/a1a3 +... +1/an-1an) =1
 x=c1+c2+ ... +cn = -1
 y=c1c2+c1c3+...+cn-1cn = 1

 c1²+c2² +... +cn² = (c1+c2+ ... +cn)² -2(c1c2+c1c3+...+cn-1cn) = -1,
 absurdo, logo para n par temos  que pelo menos 2 raízes são complexas

 []'s
 João


 --
 From: marconeborge...@hotmail.com
 To: obm-l@mat.puc-rio.br
 Subject: [obm-l] Polinômios
 Date: Wed, 25 Sep 2013 01:00:58 +

 Prove que um polinômio do tipo a(n)x^n + a(n-1)x^(n-1) + ...+ a(3)x^3 +
 x^2 + x + 1 com
 coeficientes reais não pode ter todas as raízes reais.


 --
 Esta mensagem foi verificada pelo sistema de antivírus e
 acredita-se estar livre de perigo.

 --
 Esta mensagem foi verificada pelo sistema de antivírus e
 acredita-se estar livre de perigo.

 --
 Esta mensagem foi verificada pelo sistema de antivírus e
 acredita-se estar livre de perigo.




 --
 Esdras Muniz Mota
 Graduando em Matemática Bacharelado
 Universidade Federal do Ceará


 --
 Esta mensagem foi verificada pelo sistema de antivírus e
 acredita-se estar livre de perigo.

 --
 Esta mensagem foi verificada pelo sistema de antivírus e
 acredita-se estar livre de perigo.




-- 
Esdras Muniz Mota
Graduando em Matemática Bacharelado
Universidade Federal do Ceará

Se algum dia ele recuou, foi para dar um grande salto

-- 
Esta mensagem foi verificada pelo sistema de antivírus e
 acredita-se estar livre de perigo.



[obm-l] Re: [obm-l] RE: [obm-l] Re: [obm-l] RE: [obm-l] RE: [obm-l] Polinômios

2013-09-26 Por tôpico Esdras Muniz
Obs: eu estou mostrando que as raizes de Q não podem ser todas reais, então
as de P tambem não podem.


Em 26 de setembro de 2013 11:29, Esdras Muniz
esdrasmunizm...@gmail.comescreveu:

 Q(y)=a(n) + a(n-1)y + ...+ a(3)y^(n-3) + y^(n-2) + y^(n-1) + y^n
 pelas equações de Girard, a soma das raizers é dada por menos o
 coeficiente de y^(n-1), ou seja, -1.



 Em 25 de setembro de 2013 21:14, marcone augusto araújo borges 
 marconeborge...@hotmail.com escreveu:

  Por que r1+r2+...+rn = -1?

 --
 From: esdrasmunizm...@gmail.com
 Date: Wed, 25 Sep 2013 13:28:35 -0300
 Subject: [obm-l] Re: [obm-l] RE: [obm-l] RE: [obm-l] Polinômios
 To: obm-l@mat.puc-rio.br


 Tome o polinomioQ(y)= P(1/x) fazendo y=1/x, temos:
 Q(y)=a(n) + a(n-1)y + ...+ a(3)y^(n-3) + y^(n-2) + y^(n-1) + y^n
 sendo r1, r2, ..., rn as raizes de Q(y) (com repetição). note que se R1,
 R2,..., Rn são as raizes de P(x), Ri=1/ri  (note que an é diferente de
 zero, então Q não possui raiz nula)
 Então:
 r1+r2+...+rn=-1;
 (soma sobre ij)(ri*rj)=1;
 então (r1)²+(r2)²+...+(rn)²=(r1+r2+...+rn)²-2*(soma sobre ij)(ri*rj)=
 -1 - 2*1=-3.
 Então não podemos ter todas as raízes reais.


 Em 25 de setembro de 2013 12:51, marcone augusto araújo borges 
 marconeborge...@hotmail.com escreveu:

 As expressões entre parêntesis na penúltima linha não são ambas iguais a
 1?
 E por que ´´para n par...´´?




 --
 From: joao_maldona...@hotmail.com
 To: obm-l@mat.puc-rio.br
 Subject: [obm-l] RE: [obm-l] Polinômios
 Date: Tue, 24 Sep 2013 23:00:14 -0300


 Sendo cp = 1/ap
 a1a2...an =  +-1/an
 a1a2...an(1/a1 + 1/a2+...+1/an) =   -+1/an
 a1a2...an(1/a1a2 + 1/a1a3 +... +1/an-1an) =   +-1/an

 Logo: (1/a1 + 1/a2+...+1/an) = -1
 (1/a1a2 + 1/a1a3 +... +1/an-1an) =1
 x=c1+c2+ ... +cn = -1
 y=c1c2+c1c3+...+cn-1cn = 1

 c1²+c2² +... +cn² = (c1+c2+ ... +cn)² -2(c1c2+c1c3+...+cn-1cn) = -1,
 absurdo, logo para n par temos  que pelo menos 2 raízes são complexas

 []'s
 João


 --
 From: marconeborge...@hotmail.com
 To: obm-l@mat.puc-rio.br
 Subject: [obm-l] Polinômios
 Date: Wed, 25 Sep 2013 01:00:58 +

 Prove que um polinômio do tipo a(n)x^n + a(n-1)x^(n-1) + ...+ a(3)x^3 +
 x^2 + x + 1 com
 coeficientes reais não pode ter todas as raízes reais.


 --
 Esta mensagem foi verificada pelo sistema de antivírus e
 acredita-se estar livre de perigo.

 --
 Esta mensagem foi verificada pelo sistema de antivírus e
 acredita-se estar livre de perigo.

 --
 Esta mensagem foi verificada pelo sistema de antivírus e
 acredita-se estar livre de perigo.




 --
 Esdras Muniz Mota
 Graduando em Matemática Bacharelado
 Universidade Federal do Ceará


 --
 Esta mensagem foi verificada pelo sistema de antivírus e
 acredita-se estar livre de perigo.

 --
 Esta mensagem foi verificada pelo sistema de antivírus e
 acredita-se estar livre de perigo.




 --
 Esdras Muniz Mota
 Graduando em Matemática Bacharelado
 Universidade Federal do Ceará

 Se algum dia ele recuou, foi para dar um grande salto




-- 
Esdras Muniz Mota
Graduando em Matemática Bacharelado
Universidade Federal do Ceará

Se algum dia ele recuou, foi para dar um grande salto

-- 
Esta mensagem foi verificada pelo sistema de antivírus e
 acredita-se estar livre de perigo.



[obm-l] Re: [obm-l] Re: [obm-l] Não consigo resolver

2013-12-19 Por tôpico Esdras Muniz
Não entendi o enunciado.

-- 
Esta mensagem foi verificada pelo sistema de antivírus e
 acredita-se estar livre de perigo.



[obm-l] Re: [obm-l] Re: [obm-l] Re: [obm-l] Não consigo resolver

2013-12-19 Por tôpico Esdras Muniz
Ah, agora entendi o enunciado, como o amigo ai em cima já fez a 2, a 1 vc
pode ver assim: a resposta é que n deve ser primo. Se n²|n! = n|(n-1)!,
mas um natural divide o produto de seus divisores, e se n não é primo,
todos os seus divisores aparecem no produto de (n-1)!, então n|(n-1)!.

-- 
Esta mensagem foi verificada pelo sistema de antivírus e
 acredita-se estar livre de perigo.



Re: [obm-l] Fator 2 e fator 5

2014-04-08 Por tôpico Esdras Muniz
Cara, no geral, se vc quer saber a potencia de um primo p em n!, onde np,
vc faz:
se k é o maior inteiro tal quer p^kn, então os múltiplos de p que vão
aparecer em n! são os múltiplos de p, os de p², os de p³, ..., os de p^k.
Então a maior potencia de p que divide n! é p^a, onde
a=[n/p]+[n/p²]+...+[n/p^k], onde [x]é o maior inteiro que não é maior que x
(parte inteira de x).
É claro que este k é igual a log(n) na base p. Então para o 2 é maior que
para o 5.
E alem disso, [n/2^i][n/5^i].


Em 7 de abril de 2014 21:28, Ennius Lima enn...@bol.com.br escreveu:

 Caros Colegas,

 Quando se decompõe o fatorial de n (n1) em fatores primos, parece-me que
 o fator 2 aparece mais vezes que o fator 5, pois o fator 2 está presente em
 todos os números pares, enquanto o fator 5 aparece somente nos múltiplos de
 5.
 Gostaria de saber como podemos provar isso formalmente.

 Desde já, muito obrigado.
 Ennius Lima
 ___


 --
 Esta mensagem foi verificada pelo sistema de antivírus e
  acredita-se estar livre de perigo.




-- 
Esdras Muniz Mota
Graduando em Matemática Bacharelado
Universidade Federal do Ceará

-- 
Esta mensagem foi verificada pelo sistema de antivírus e
 acredita-se estar livre de perigo.



[obm-l] Re: [obm-l] Provar que D = {x | f(x-) =! f(x+)} é enumerável

2014-10-14 Por tôpico Esdras Muniz
A ideia é a seguinte, vou fazer com uma função particular mas pode ser
adaptado para o caso geral:
vamos tomas a função que assume apenas os valores 0 ou 1. Se o limite de x
tendendo a t pela esquerda é 1, então existe um e(t)0 tq se x pertence a
(t-e(t), t) então f(x)=1. Então suponha que o conjunto  D das
descontinuidades de f seja não enumerável, temos então que a soma dos e(t)
com t pertencente a D é no máximo 1. Isto gera absurdo, pois é fácil provar
que a soma de uma quantidade não enumerável de números positivos não pode
ser finita. Agora basta fazer algumas adaptações e provar alguns fatos.

Em 13 de outubro de 2014 20:01, Amanda Merryl sc...@hotmail.com escreveu:

 Oi amigos, podem ajudar nisto aqui?

 Seja f uma função real definida em (a, b) e D o conjunto dos pontos de
 (a, b) no qual f apresenta descontinuidade do tipo salto (os limites Ã
 direita e à esquerda existem em R e são diferentes). Mostre que D é
 enumerável.

 Obrigada

 Amanda



 --
 Esta mensagem foi verificada pelo sistema de antivírus e
  acredita-se estar livre de perigo.


 =
 Instruções para entrar na lista, sair da lista e usar a lista em
 http://www.mat.puc-rio.br/~obmlistas/obm-l.html
 =




-- 
Esdras Muniz Mota
Graduando em Matemática Bacharelado
Universidade Federal do Ceará

-- 
Esta mensagem foi verificada pelo sistema de antiv�rus e
 acredita-se estar livre de perigo.



[obm-l] Re: [obm-l] Somatório

2014-10-22 Por tôpico Esdras Muniz
Dá 41.

Em 21 de outubro de 2014 19:53, ruymat...@ig.com.br escreveu:

  Não lembro a notação para somatório usada aqui. Vou escrever assim: Seja
 o SOMATÓRIO com n variando de zero a infinito de
 sen(nx)/3^n=(a+bsqrt(2))/c. Se mdc(a,b)=1 , senx=1/3 e 0=x=pi/2, calcule
 a+b+c. Quem ajudar, agradeço antecipadamente. Abraços a todos.

 --
 Esta mensagem foi verificada pelo sistema de antivírus e
 acredita-se estar livre de perigo.




-- 
Esdras Muniz Mota
Graduando em Matemática Bacharelado
Universidade Federal do Ceará

-- 
Esta mensagem foi verificada pelo sistema de antiv�rus e
 acredita-se estar livre de perigo.



[obm-l] Re: [obm-l] Somatório

2014-10-22 Por tôpico Esdras Muniz
É só usar a forma complexa do seno e transformar na diferença de duas
séries geométricas. Aí a soma dá (5+2sqrt(2))/34

Em 22 de outubro de 2014 10:02, Esdras Muniz esdrasmunizm...@gmail.com
escreveu:

 Dá 41.

 Em 21 de outubro de 2014 19:53, ruymat...@ig.com.br escreveu:

  Não lembro a notação para somatório usada aqui. Vou escrever assim: Seja
 o SOMATÓRIO com n variando de zero a infinito de
 sen(nx)/3^n=(a+bsqrt(2))/c. Se mdc(a,b)=1 , senx=1/3 e 0=x=pi/2, calcule
 a+b+c. Quem ajudar, agradeço antecipadamente. Abraços a todos.

 --
 Esta mensagem foi verificada pelo sistema de antivírus e
 acredita-se estar livre de perigo.




 --
 Esdras Muniz Mota
 Graduando em Matemática Bacharelado
 Universidade Federal do Ceará





-- 
Esdras Muniz Mota
Graduando em Matemática Bacharelado
Universidade Federal do Ceará

-- 
Esta mensagem foi verificada pelo sistema de antiv�rus e
 acredita-se estar livre de perigo.



Re: [obm-l] Primo e divisibilidade

2014-10-28 Por tôpico Esdras Muniz
Lema: se p=3(mod4) e p | a²+b² então p | a e p | b.
p=4k+3, suponha p não divide a e p não divide b.
por Fermat a^(4k+2)=1(mod p) e b^(4k+2)=1(mod p) = a^(4k+2)=b^(4k+2) (mod
p) (i)
mas como p | a²+b² = a²=b²(mod p) elevando a (2k+1):
a^(4k+2)=((-1)^(2k+1))*b^(4k+2)(mod p) = a^(4k+2)= -b^(4k+2)(mod p) (ii)
(i) e (ii) geram absurdo, e o lema está provado.

Em 25 de outubro de 2014 11:05, marcone augusto araújo borges 
marconeborge...@hotmail.com escreveu:

 Seja p um número primo ímpar. Mostre que se p divide a^2 + b^2 com (a,b) =
 1, então
 p = 1 (mod 4).

 --
 Esta mensagem foi verificada pelo sistema de antivírus e
 acredita-se estar livre de perigo.




-- 
Esdras Muniz Mota
Graduando em Matemática Bacharelado
Universidade Federal do Ceará

-- 
Esta mensagem foi verificada pelo sistema de antiv�rus e
 acredita-se estar livre de perigo.



Re: [obm-l] ajuda para atacar este problema

2014-10-28 Por tôpico Esdras Muniz
6° problema da OBMU, só percebi q α é irracional. Tava pensando que poderia
ser feito dividindo o [0,1] como [0,1/N]; [1/N,2/N];...; [(N-1)/N,1]
e mostrando que tem um elemento do X em cada parte.

Em 28 de outubro de 2014 17:05, Bruno Rodrigues 
brunorodrigues@gmail.com escreveu:

 Oi pessoal,estou sem ideias para este problema:

 Considere um número real α e constantes b  0 e γ ≥ 1 tais que para
 quaisquer p e q inteiros com q ≥ 1 vale
 |qα − p| ≥ b/qγ.
 Prove que existe uma constante C tal que, para todo inteiro N ≥ 1, o
 conjunto

 XN = {mα − ɭmα⌡, m ∈ Z, 0 ≤ m ≤ CNγ}
 é tal que, para todo x ∈ [0, 1] existe y ∈ XN com |x − y|  1/N.

 nota: ɭmα⌡ é a parte inteira de mα.

 Alguem tem alguma sugestao de como desenvolver uma bom raciocinio para ela?
 Como voces a atacariam?

 Abraços


 --
 Esta mensagem foi verificada pelo sistema de antivírus e
 acredita-se estar livre de perigo.




-- 
Esdras Muniz Mota
Graduando em Matemática Bacharelado
Universidade Federal do Ceará

-- 
Esta mensagem foi verificada pelo sistema de antiv�rus e
 acredita-se estar livre de perigo.



[obm-l] Re: [obm-l] Re: Convergência ou divergência de sequência e de séries

2014-10-30 Por tôpico Esdras Muniz
a) Basta usar q: Soma(k = 1, n) f(k)Int [1, n] f(x) Soma(k = 1, n+1).


Em 30 de outubro de 2014 08:14, Amanda Merryl sc...@hotmail.com escreveu:

 Oi Artur

 Na sua resposta só veio o problema original e seu nome.

 Amanda


  Em 30/10/2014, às 09:11, Amanda Merryl sc...@hotmail.com escreveu:
 
  Bom  dia. Estou com alguma dificuldade nisto. Agradeço se puderem ajudar
 em um deles.
 
  a) Seja f:[1, oo) decrescente e limitada e seja (a_n) dada por
 
  a_n = Soma(k = 1, n) f(k) - Int [1, n] f(x) dx, n = 1, 2,3 .
 
  Mostre que (a_n) converge (mesmo que a série e a integral divirjam. Em
 caso de convergência de ambas, o resultado é imediato. Aliás, pelo teste da
 integral, ou ambas convergem ou ambas divergem)
 
  b) Seja (a_n) uma sequência de reais positivos e (s_n) a sequência de
 suas somas parciais. Estude a convergência/divergência de Soma (a_n)/(s_n)
 para os seguintes casos:
 
  b.1) a_n = 1/n^2, n = 1, 2, 3
 
  b.2) a_n = 1/(p_n), sendo p_n o n-gésimo primo.
 
  Muito obrigada
 
  Amanda.
 
  Artur Costa Steiner

 --
 Esta mensagem foi verificada pelo sistema de antivírus e
  acredita-se estar livre de perigo.


 =
 Instru�ões para entrar na lista, sair da lista e usar a lista em
 http://www.mat.puc-rio.br/~obmlistas/obm-l.html
 =




-- 
Esdras Muniz Mota
Graduando em Matemática Bacharelado
Universidade Federal do Ceará

-- 
Esta mensagem foi verificada pelo sistema de antiv�rus e
 acredita-se estar livre de perigo.



[obm-l] Re: [obm-l] OBM NÍVEL 3 TERCEIRA FASE PRIMEIRO DIA

2014-10-30 Por tôpico Esdras Muniz
Opa, eu tinha entendido círculos circunscritos... Foi mal.

Em 30 de outubro de 2014 11:02, Esdras Muniz esdrasmunizm...@gmail.com
escreveu:



 Em 29 de outubro de 2014 22:50, Douglas Oliveira de Lima 
 profdouglaso.del...@gmail.com escreveu:


  *PROBLEMA 1 *

 Seja *ABCD *um quadrilátero convexo e seja *P *a interseção das
 diagonais *AC *e *BD*. Os raios dos círculos inscritos nos triângulos
 *ABP*, *BCP*, *CDP *e *DAP *são iguais. Prove que *ABCD *é um losango.


 Como poderíamos fazer esse problema?




 --
 Esta mensagem foi verificada pelo sistema de antivírus e
 acredita-se estar livre de perigo.




 --
 Esdras Muniz Mota
 Graduando em Matemática Bacharelado
 Universidade Federal do Ceará





-- 
Esdras Muniz Mota
Graduando em Matemática Bacharelado
Universidade Federal do Ceará

-- 
Esta mensagem foi verificada pelo sistema de antiv�rus e
 acredita-se estar livre de perigo.



[obm-l] Re: [obm-l] Re: [obm-l] Indução

2014-11-17 Por tôpico Esdras Muniz
Um problema legal relacionado com este é o seguinte:
Calcule a cardinalidade do conjunto C={ax-by | x,y ∈N}∩N onde N={1, 2, 3,
...} Onde a e b são naturais dados.

Resposta: (a-1)(b-1)/2.

Em 17 de novembro de 2014 08:35, Ralph Teixeira ralp...@gmail.com
escreveu:

 Seja P(n): o banco pode pagar a quantia de n reais.

 Então:
 P(8) é verdadeira: 8=3+5
 P(9) é verdadeira: 9=3+3+3
 P(10) é verdadeira: 10=5+5

 Agora, se P(k) é verdadeira, então P(k+3) também é.
 De fato, basta pagar k reais da maneira que é possível, e adicionar uma
 nota de $3.

 Por indução, P(n) vale para todo n=8.

 ---///---

 Essa foi uma indução de passo 3. Se você quiser converter isso numa
 indução de passo 1, use:
 Q(n): o banco pode pagar n, n+1 e n+2 reais.

 Então:
 i) Q(8) é verdadeira (vide P(8), P(9) e P(10) acima).
 ii) Se Q(k) é verdadeira, Q(k+1) também é.
 (Pois se pode pagar k, k+1 e k+2, então obviamente pode pagar k+1 e k+2.
 Para pagar k+3, pague k e ponha uma nota de 3.)

 Por indução, Q(n) é verdadeira para todo n=8.

 Abraço,
 Ralph

 2014-11-15 9:19 GMT-02:00 marcone augusto araújo borges 
 marconeborge...@hotmail.com:

 Em um país longinquo, a moeda local é o cruzeiro.Neste país um banco tem
 uma quantidade ilimitada
 de cédulas de 3 e 5 crzeiros.Prove, por indução, que o banco pode pagar
 uma quantidade qualquer(inteira)
 de cruzeiros, maior que 7


 --
 Esta mensagem foi verificada pelo sistema de antivírus e
 acredita-se estar livre de perigo.



 --
 Esta mensagem foi verificada pelo sistema de antivírus e
 acredita-se estar livre de perigo.




-- 
Esdras Muniz Mota
Graduando em Matemática Bacharelado
Universidade Federal do Ceará

-- 
Esta mensagem foi verificada pelo sistema de antiv�rus e
 acredita-se estar livre de perigo.



Re: [obm-l] Teorema

2014-12-19 Por tôpico Esdras Muniz
Dá uma olhada no postulado de Bertrand.

Em 19 de dezembro de 2014 16:44, Marcos Martinelli mffmartine...@gmail.com
escreveu:

 Sim. Complicada. Decorre de um teorema de Chebyshev.

 2014-12-19 17:32 GMT-02:00 marcone augusto araújo borges 
 marconeborge...@hotmail.com:

 Seja p um primo.Existe um primo p´tal que p  p´ 2p.
 A demostração é complicada?Onde achar?

 --
 Esta mensagem foi verificada pelo sistema de antivírus e
 acredita-se estar livre de perigo.


 --
 Esta mensagem foi verificada pelo sistema de antivírus e
 acredita-se estar livre de perigo.



-- 
Esdras Muniz Mota
Graduando em Matemática Bacharelado
Universidade Federal do Ceará

-- 
Esta mensagem foi verificada pelo sistema de antiv�rus e
 acredita-se estar livre de perigo.



[obm-l] Re: [obm-l] Centro da circunferência

2015-01-06 Por tôpico Esdras Muniz
Diga!os que o comprimento das cordas seja l, então P divide l em duas
partes de comprimentos x e y, assim, x.y seria a potencia de P e  a some de
x e y seria l. Dai vc tira que existem dois nu!eros x e y que dependem de l
e P, que são invariantes. Agora sejam A, B e C os pontos da circunferência
distando x de p, então P seria o circuncentro do triangulo ABC.
Em 06/01/2015 12:55, Carlos Gomes cgomes...@gmail.com escreveu:

 Olá amigos,

 Algum de você pode me ajudar com essa questão:

 Seja P um ponto no interior de um círculo tal que existem três cordas que
 passam
 por P e tem o mesmo comprimento. Prove que P é o centro do círculo.

 Grato, Cgomes.

 --
 Esta mensagem foi verificada pelo sistema de antivírus e
 acredita-se estar livre de perigo.

-- 
Esta mensagem foi verificada pelo sistema de antiv�rus e
 acredita-se estar livre de perigo.



Re: [obm-l] Geometria

2015-03-06 Por tôpico Esdras Muniz
Agora vim ver q vc queria sem a lei dos senos.

Em 6 de março de 2015 20:20, Douglas Oliveira de Lima 
profdouglaso.del...@gmail.com escreveu:

 Nao consegui concluir dessa forma.

 Em 6 de março de 2015 19:30, Esdras Muniz esdrasmunizm...@gmail.com
 escreveu:


 Em 6 de março de 2015 19:30, Esdras Muniz esdrasmunizm...@gmail.com
 escreveu:

 Tome E pertencente ao lado AB, tal que o ângulo BDE vale 10°, daí trace
 as retes DE e EC, marque os ângulos e conclua.

 Em 6 de março de 2015 19:06, Douglas Oliveira de Lima 
 profdouglaso.del...@gmail.com escreveu:

 Olá, será que existe uma solução por traçados para seguinte questao:
 Dado um triângulo isósceles ABC com AB=AC,  e um ponto D no lado AC tal
 que AD=BC,  e o ângulo ABD vale 10 graus,  achar o ângulo BAC.

 Douglas oliveira

 --
 Esta mensagem foi verificada pelo sistema de antivírus e
 acredita-se estar livre de perigo.




 --
 Esdras Muniz Mota
 Mestrando em Matemática
 Universidade Federal do Ceará





 --
 Esdras Muniz Mota
 Mestrando em Matemática
 Universidade Federal do Ceará



 --
 Esta mensagem foi verificada pelo sistema de antivírus e
 acredita-se estar livre de perigo.



 --
 Esta mensagem foi verificada pelo sistema de antivírus e
 acredita-se estar livre de perigo.




-- 
Esdras Muniz Mota
Mestrando em Matemática
Universidade Federal do Ceará

-- 
Esta mensagem foi verificada pelo sistema de antiv�rus e
 acredita-se estar livre de perigo.



Re: [obm-l] Geometria

2015-03-06 Por tôpico Esdras Muniz
Em 6 de março de 2015 19:30, Esdras Muniz esdrasmunizm...@gmail.com
escreveu:

 Tome E pertencente ao lado AB, tal que o ângulo BDE vale 10°, daí trace as
 retes DE e EC, marque os ângulos e conclua.

 Em 6 de março de 2015 19:06, Douglas Oliveira de Lima 
 profdouglaso.del...@gmail.com escreveu:

 Olá, será que existe uma solução por traçados para seguinte questao:
 Dado um triângulo isósceles ABC com AB=AC,  e um ponto D no lado AC tal
 que AD=BC,  e o ângulo ABD vale 10 graus,  achar o ângulo BAC.

 Douglas oliveira

 --
 Esta mensagem foi verificada pelo sistema de antivírus e
 acredita-se estar livre de perigo.




 --
 Esdras Muniz Mota
 Mestrando em Matemática
 Universidade Federal do Ceará





-- 
Esdras Muniz Mota
Mestrando em Matemática
Universidade Federal do Ceará

-- 
Esta mensagem foi verificada pelo sistema de antiv�rus e
 acredita-se estar livre de perigo.



Re: [obm-l] Geometria

2015-03-06 Por tôpico Esdras Muniz
Tome E pertencente ao lado AB, tal que o ângulo BDE vale 10°, daí trace as
retes DE e EC, marque os ângulos e conclua.

Em 6 de março de 2015 19:06, Douglas Oliveira de Lima 
profdouglaso.del...@gmail.com escreveu:

 Olá, será que existe uma solução por traçados para seguinte questao:
 Dado um triângulo isósceles ABC com AB=AC,  e um ponto D no lado AC tal
 que AD=BC,  e o ângulo ABD vale 10 graus,  achar o ângulo BAC.

 Douglas oliveira

 --
 Esta mensagem foi verificada pelo sistema de antivírus e
 acredita-se estar livre de perigo.




-- 
Esdras Muniz Mota
Mestrando em Matemática
Universidade Federal do Ceará

-- 
Esta mensagem foi verificada pelo sistema de antiv�rus e
 acredita-se estar livre de perigo.



Re: [obm-l] Prove que...

2015-03-27 Por tôpico Esdras Muniz
Faz por indução, vc supõe q o resultado vale para (n-1).
Divida o comj de 2^(n+1) naturais em dois de igual tamanho, 2^n.
Por HI, cada um tem um subconj de tamanho 2^(n-1) cuja soma é múltipla de
2^(n-1), digamos que essas somas são S1 e S2.
Fora os números usados em S1 e S2, restam ainda 2^n naturais, e novamente
por HI temos um subconj de tamanho 2^(n-1) cuja soma S3 é múltipla de
2^(n-1).

Se S1=2^(n-1)*a, S2=2^(n-1)*b e S3=2^(n-1)*c.
Dentre os números a, b e c, há dois pares ou dois ímpares, Então há duas
das somas cuha soma é múltipla de 2^n.

Em 26 de março de 2015 22:44, Cassio Anderson Feitosa 
cassiofeito...@gmail.com escreveu:

 Ele quis dizer que se forem dados 2^(n+1) naturais, é possível escolher
 2^n desses naturais de modo que a soma deles seja divisivel opr 2^n.

 Em 26 de março de 2015 22:23, Douglas Oliveira de Lima 
 profdouglaso.del...@gmail.com escreveu:

 Entre o que?
  Em 26/03/2015 21:33, marcone augusto araújo borges 
 marconeborge...@hotmail.com escreveu:

 Prove que, entre 2^(n+1) números naturais quaisquer, existem 2^n números
 cuja soma
 é divisível por 2^n



 --
 Esta mensagem foi verificada pelo sistema de antivírus e
 acredita-se estar livre de perigo.


 --
 Esta mensagem foi verificada pelo sistema de antivírus e
 acredita-se estar livre de perigo.




 --
 Cássio Anderson
 Graduando em Matemática - UFPB

 --
 Esta mensagem foi verificada pelo sistema de antivírus e
 acredita-se estar livre de perigo.




-- 
Esdras Muniz Mota
Mestrando em Matemática
Universidade Federal do Ceará

-- 
Esta mensagem foi verificada pelo sistema de antiv�rus e
 acredita-se estar livre de perigo.



Re: [obm-l] Prove que...

2015-03-27 Por tôpico Esdras Muniz
Eu queria ver a prova por função geratriz :)

Em 27 de março de 2015 10:41, Esdras Muniz esdrasmunizm...@gmail.com
escreveu:

 Faz por indução, vc supõe q o resultado vale para (n-1).
 Divida o comj de 2^(n+1) naturais em dois de igual tamanho, 2^n.
 Por HI, cada um tem um subconj de tamanho 2^(n-1) cuja soma é múltipla de
 2^(n-1), digamos que essas somas são S1 e S2.
 Fora os números usados em S1 e S2, restam ainda 2^n naturais, e novamente
 por HI temos um subconj de tamanho 2^(n-1) cuja soma S3 é múltipla de
 2^(n-1).

 Se S1=2^(n-1)*a, S2=2^(n-1)*b e S3=2^(n-1)*c.
 Dentre os números a, b e c, há dois pares ou dois ímpares, Então há duas
 das somas cuha soma é múltipla de 2^n.

 Em 26 de março de 2015 22:44, Cassio Anderson Feitosa 
 cassiofeito...@gmail.com escreveu:

 Ele quis dizer que se forem dados 2^(n+1) naturais, é possível escolher
 2^n desses naturais de modo que a soma deles seja divisivel opr 2^n.

 Em 26 de março de 2015 22:23, Douglas Oliveira de Lima 
 profdouglaso.del...@gmail.com escreveu:

 Entre o que?
  Em 26/03/2015 21:33, marcone augusto araújo borges 
 marconeborge...@hotmail.com escreveu:

 Prove que, entre 2^(n+1) números naturais quaisquer, existem 2^n números
 cuja soma
 é divisível por 2^n



 --
 Esta mensagem foi verificada pelo sistema de antivírus e
 acredita-se estar livre de perigo.


 --
 Esta mensagem foi verificada pelo sistema de antivírus e
 acredita-se estar livre de perigo.




 --
 Cássio Anderson
 Graduando em Matemática - UFPB

 --
 Esta mensagem foi verificada pelo sistema de antivírus e
 acredita-se estar livre de perigo.




 --
 Esdras Muniz Mota
 Mestrando em Matemática
 Universidade Federal do Ceará





-- 
Esdras Muniz Mota
Mestrando em Matemática
Universidade Federal do Ceará

-- 
Esta mensagem foi verificada pelo sistema de antiv�rus e
 acredita-se estar livre de perigo.



Re: [obm-l] Probabilidade Random quadratic equations

2015-03-03 Por tôpico Esdras Muniz
Ah, verdade, fui fazer de cabeça e errei XD.

Em 3 de março de 2015 12:59, Pedro José petroc...@gmail.com escreveu:

 Boa tarde!
 Na verdade  ∆ = 4.r^2 - 4s.
 s =0 ==  ∆= 0 para todo r, logo já saímos de 1/2.

 s 0 :  ∆= 0 == |r|= raiz(s)

 A probabilidade de |r| = raiz(s), que, para meu conhecimento, é difícil
 de caracterizar (embora intuitivamente creia que seja 1/2). Porém vavos
 chamá-la de p'.

 p = 1/2 + 1/2 * p'; e eu chutaria 3/4


 Saudações,
 PJMS.

 Em 3 de março de 2015 11:22, Esdras Muniz esdrasmunizm...@gmail.com
 escreveu:

 Vc faz delta=0 e obtém |r|=|s| e analisando o gráfico vê que a
 probabilidade é 1/2.

 Em 3 de março de 2015 10:55, Douglas Oliveira de Lima 
 profdouglaso.del...@gmail.com escreveu:

 Olá caros amigos, recebi um problema esta semana, e gostaria de uma ajuda
 (se possível), dos senhores.
 Pesquisei esse problema na internet e achei algumas divergências de
 soluções.
 Eis o problema:
 Qual a probabilidade da equação do segundo grau x^2+2rx+s=0 ter raiz
 real?



 Agradeço desde já a ajuda.
 Douglas Oliveira.


 --
 Esta mensagem foi verificada pelo sistema de antivírus e
 acredita-se estar livre de perigo.




 --
 Esdras Muniz Mota
 Mestrando em Matemática
 Universidade Federal do Ceará



 --
 Esta mensagem foi verificada pelo sistema de antivírus e
 acredita-se estar livre de perigo.



 --
 Esta mensagem foi verificada pelo sistema de antivírus e
 acredita-se estar livre de perigo.




-- 
Esdras Muniz Mota
Mestrando em Matemática
Universidade Federal do Ceará

-- 
Esta mensagem foi verificada pelo sistema de antiv�rus e
 acredita-se estar livre de perigo.



[obm-l] Re: [obm-l] Romênia

2015-02-27 Por tôpico Esdras Muniz
Bem, para a bijeção só falta mostrar a injeção, suponha por absurdo xy e
f(x)=f(y), a sequência x, y, x, y, x, y,  é divergente, mas sua imagem
não, pois é constante, já q f(x)=f(y).
Agora, suponha a inversa g descontínua, então existe e0, e x real tais
que para todo n natural,
|g(x)-g(y)|e, para |x-y|1/n. Então vc faz x=f(a) e y=f(bn), onde a
sequência bn é divergente, assim fica:
|a-bn|e (já que bn diverge) além disso |f(a)-f(bn)|1/n, o que implica que
f(bn) converge para f(a), gerando um absurdo.
Talvez haja algum erro bobo que precise ser corrigido, mas acho q é isso.

Em 27 de fevereiro de 2015 08:37, Gabriel Lopes cronom...@gmail.com
escreveu:

 *Gostaria de ajuda com a seguinte questão vinda da Romênia , acho que da
 olimpíada (o livro não especifica qual olímpíada e qual ano) :

 - Seja f: R -- R  uma função sobrejetiva , satisfazendo a seguinte
 propriedade : para toda sequência divergente (a_n) , n  = 1 ,  a sequência
 (f(a_n)) , n = 1 , também é divergente .  Prove que  f  é bijetiva e que
 sua função inversa f^(-1) é contínua.

 *O livro oferta a seguintes dicas :

 1.(Para provar que f é bijetiva) Tome x,y  em R distintos e considere
 (a_n) ,n  = 1 , a sequência divergente tal que a_2k = x  e  a_2k-1 = y ,
 para todo k  = 1 , e utilize a segunda hipótese do enunciado .

 * Aqui deduzimos que existe e  0  tal que  para todo n* em N  temos: m,n
  n*,  m  n ,  então   | f(a_m) - f(a_n) | = | f(x) - f(y) |  =  e  ;
 pela relação entre sequências convergentes e  sequências de Cauchy ,  e
 então negando a afirmação :  ( f(a_n) ) ,n  =1 ,  é convergente .

 2.(Para provar que f^(-1)  é contínua) Use as Hipóteses sobre f  para
 mostrar que f^(-1) transforma sequências convergentes em sequências
 convergentes.

 *Parei por aqui mas os seguintes comentários são pertinentes :

 I.  O  capítulo do livro em que tirei este problema fala sobre
 Continuidade Sequencial  e prova  o seguinte TMA : Uma função f: I -- R
 , onde I é um intervalo, é contínua se , e só se , a seguinte condição for
 satisfeita : para todo a  em  I   e  toda sequência (a_n),n  = 1, de
 elementos de I ,  temos : lim(a_n)  = a   , então  lim( f(a_n) ) =  f(a)
 .  (não consegui só com ele)

 II. Procurei sobre o TMA em outro livro ,Curso de Análise Vol.1  Elon
 Lages , Capítulo VII , e encontrei o seguinte corolário :  A fim de que f
 seja contínua  no ponto  a  , é suficiente que , para toda sequência  de
 pontos a_n  de  X   ( creio  que  X  é uma união de Intervalos)   com  lim(
 a_n )  =  a   ,   exista   lim( f(a_n) ) .  O mesmo não foi demonstrado
 ,e  também não consegui faze-lo  , mas acho que ele é suficiente para
 resolver a questão.


 --
 Esta mensagem foi verificada pelo sistema de antivírus e
 acredita-se estar livre de perigo.




-- 
Esdras Muniz Mota
Mestrando em Matemática
Universidade Federal do Ceará

-- 
Esta mensagem foi verificada pelo sistema de antiv�rus e
 acredita-se estar livre de perigo.



Re: [obm-l] Probabilidade Random quadratic equations

2015-03-03 Por tôpico Esdras Muniz
Vc faz delta=0 e obtém |r|=|s| e analisando o gráfico vê que a
probabilidade é 1/2.

Em 3 de março de 2015 10:55, Douglas Oliveira de Lima 
profdouglaso.del...@gmail.com escreveu:

 Olá caros amigos, recebi um problema esta semana, e gostaria de uma ajuda
 (se possível), dos senhores.
 Pesquisei esse problema na internet e achei algumas divergências de
 soluções.
 Eis o problema:
 Qual a probabilidade da equação do segundo grau x^2+2rx+s=0 ter raiz real?



 Agradeço desde já a ajuda.
 Douglas Oliveira.


 --
 Esta mensagem foi verificada pelo sistema de antivírus e
 acredita-se estar livre de perigo.




-- 
Esdras Muniz Mota
Mestrando em Matemática
Universidade Federal do Ceará

-- 
Esta mensagem foi verificada pelo sistema de antiv�rus e
 acredita-se estar livre de perigo.



[obm-l] Re: [obm-l] Re: [obm-l] Ajuda - OBM 2014 nível universitário

2015-02-23 Por tôpico Esdras Muniz
Cara, faz tempo isso, mas fiz por volume, vc usa que o tetraedro de maior
volume inscrito na esfera é o regular.

Em 22 de fevereiro de 2015 22:26, Ralph Teixeira ralp...@gmail.com
escreveu:

 Acho que a culpa dessa expressao eh minha -- eu tenho essa mania de chamar
 funcoes afins de lineares, vem do ingles (linear functions).

 Linear em cada entrada quer dizer o seguinte: se voce fixar todas as
 entradas exceto uma, digamos, a_11=x, a funcao determinante seria f(x)=ax+b
 onde a e b dependem apenas das outras 8 entradas... Entao, fixadas as
 outras 8 entradas, a funcao f(x) serah maximizada em x=0 ou x=9 (bom, pode
 ser que a=0, entao qualquer valor de x daria no mesmo, mas voce nao perde
 nada em supor x=0 ou x=9). Entao nao eh que x TEM que ser 0 ou 9, eh que
 voce PODE supor x=0 ou x=9 para maximizar a funcao. Como isso vale para
 cada uma das 9 entradas...

 Melhorou?

 Abraco, Ralph.

 P.S.: Ou talvez, pense por contradicao: se det(A) fosse maximizado com
 alguma entrada NAO sendo 0 ou 9, voce poderia trocar esta entrada para 0 ou
 9 e isto aumentaria (ou manteria) o valor do determinante, Entao HA uma
 escolha maximizante apenas com 0 ou 9.

 2015-02-22 14:14 GMT-05:00 João Maldonado joao_maldona...@hotmail.com:

 Fala ai gente,

 Fiquei com uma dúvida no problema 2 da OBM-2014 nível universitário,
 primeira fase. Tentei resolver o problema, não consegui, quado fui olhar a
 resolução me perdi logo nas primeiras linhas, teria como alguém me dar uma
 ajuda?

 O problema é o seguinte: Considere as matrizes 3x3 cujas entradas são
 inteiros entre 0 e 9 (inclusive). Determine o maior determinante possível
 de uma tal matriz.

 A resolução começa assim:

 Seja A = (aij) a matriz. Como det(A) é linear em cada entrada, basta
 considerar aij = 0 ou aij = 9, de modo que A = 9B com B = (bij ) e bij = 0
 ou 1.

 Não entendi o que ele quis dizer como linear em cada entrada. Teria
 como alguém me explicar melhor porque os valores só podem ser 9 ou 0?

 []'s
 João

 --
 Esta mensagem foi verificada pelo sistema de antivírus e
 acredita-se estar livre de perigo.



 --
 Esta mensagem foi verificada pelo sistema de antivírus e
 acredita-se estar livre de perigo.




-- 
Esdras Muniz Mota
Mestrando em Matemática
Universidade Federal do Ceará

-- 
Esta mensagem foi verificada pelo sistema de antiv�rus e
 acredita-se estar livre de perigo.



[obm-l] Re: [obm-l] Sequências

2015-02-25 Por tôpico Esdras Muniz
Se tn=kt(n+1) então o termo de ordem k é n.

Em 25 de fevereiro de 2015 16:09, Jefferson Franca jeffma...@yahoo.com.br
escreveu:

 Boa tarde para todos. Um aluno me enviou este problema que não consigo
 resolver: Juquinha gosta de diversões matemáticas, uma delas consiste em
 descobrir números de sequências. Por exemplo,
 1,2,2,3,3,3,4,4,4,4,5,5,5,5,5,..., onde cada número natural n é escrito n
 vezes. Determine o número de ordem 1000.
 Será que alguém aqui saberia elucidar este mistério?
 Att
 Jefferson

 --
 Esta mensagem foi verificada pelo sistema de antivírus e
 acredita-se estar livre de perigo.




-- 
Esdras Muniz Mota
Mestrando em Matemática
Universidade Federal do Ceará

-- 
Esta mensagem foi verificada pelo sistema de antiv�rus e
 acredita-se estar livre de perigo.



[obm-l] Re: [obm-l] Sequências

2015-02-25 Por tôpico Esdras Muniz
Sendo tn o n-esimo número triangular.

Em 25 de fevereiro de 2015 16:44, Esdras Muniz esdrasmunizm...@gmail.com
escreveu:

 Se tn=kt(n+1) então o termo de ordem k é n.

 Em 25 de fevereiro de 2015 16:09, Jefferson Franca jeffma...@yahoo.com.br
  escreveu:

 Boa tarde para todos. Um aluno me enviou este problema que não consigo
 resolver: Juquinha gosta de diversões matemáticas, uma delas consiste em
 descobrir números de sequências. Por exemplo,
 1,2,2,3,3,3,4,4,4,4,5,5,5,5,5,..., onde cada número natural n é escrito n
 vezes. Determine o número de ordem 1000.
 Será que alguém aqui saberia elucidar este mistério?
 Att
 Jefferson

 --
 Esta mensagem foi verificada pelo sistema de antivírus e
 acredita-se estar livre de perigo.




 --
 Esdras Muniz Mota
 Mestrando em Matemática
 Universidade Federal do Ceará





-- 
Esdras Muniz Mota
Mestrando em Matemática
Universidade Federal do Ceará

-- 
Esta mensagem foi verificada pelo sistema de antiv�rus e
 acredita-se estar livre de perigo.



Re: [obm-l] Como provar?

2015-03-27 Por tôpico Esdras Muniz
Onde encontro essa solução?


Em 27 de março de 2015 13:38, Eduardo Henrique dr.dhe...@outlook.com
escreveu:

 Ah, somatório de 2 elevado a i, com indice i nos naturais. Na verdade eu
 escrevi menos do que eu deveria, pois na verdade temos que é um somatório
 de alpha sub-indice i vezes 2^i, o índice i pertencente aos naturais.

 --
 From: marconeborge...@hotmail.com
 To: obm-l@mat.puc-rio.br
 Subject: [obm-l] Como provar?
 Date: Fri, 27 Mar 2015 12:23:41 +


 Eduardo, o que significa sum_ i 2^i ?

 --
 Esta mensagem foi verificada pelo sistema de antivírus e
 acredita-se estar livre de perigo.

 --
 Esta mensagem foi verificada pelo sistema de antivírus e
 acredita-se estar livre de perigo.




-- 
Esdras Muniz Mota
Mestrando em Matemática
Universidade Federal do Ceará

-- 
Esta mensagem foi verificada pelo sistema de antiv�rus e
 acredita-se estar livre de perigo.



Re: [obm-l] Problema das caixas

2015-04-22 Por tôpico Esdras Muniz
Acho q é 1169.

Em 21 de abril de 2015 15:21, Mariana Groff bigolingroff.mari...@gmail.com
escreveu:

 Boa tarde,
 Alguém poderia me ajudar no problema a seguir?

 Temos 27 caixas em fila; cada uma delas contém pelo menos 12 bolinhas. A
 operação permitida é transferir uma bolinha de uma caixa para sua vizinha
 da direita, se essa vizinha da direita tem mais bolinhas. Dizemos que uma
 distribuição inicial das bolinhas é *feliz* se é possível, mediante uma
 sucessão de operações permitidas, fazer com que todas as bolinhas fiquem
 numa mesma caixa. Determine o menor número total de bolinhas de uma
 distribuição inicial feliz.

 Obrigada,
 Mariana

 --
 Esta mensagem foi verificada pelo sistema de antivírus e
 acredita-se estar livre de perigo.




-- 
Esdras Muniz Mota
Mestrando em Matemática
Universidade Federal do Ceará

-- 
Esta mensagem foi verificada pelo sistema de antiv�rus e
 acredita-se estar livre de perigo.



Re: [obm-l] irracionalidade

2015-04-29 Por tôpico Esdras Muniz
Algébrico é o número que é raiz de algum polinômio não identicamente nulo e
de coeficientes inteiros
Por exemplo (1/2)^1/2, é raix do polinômio p(x)=2x²-1. Os reais que não são
algébricos são chamados transcendentes.

Em 29 de abril de 2015 17:31, Listeiro 037 listeiro_...@yahoo.com.br
escreveu:


 Olá.

 (sqrt(3))^3 = 3*sqrt(3) (irracional)
 (sqrt(3+1))^3 = 8 (racional)

 Este contra-exemplo é bom. Então, não seria transcendental?

 Transcendental é tipo e=2,7181... PI=3,141592... diferente de raiz
 quadrada de 2 que é raiz da equação de termos finitos  x^2-2=0; peço
 prá alguém com mais traquejo defina transcendental/algébrico porque
 posso não ser exato.

 Se r^k é transcendental, então (r+1)^k também é transcendental?

 Em Wed, 29 Apr 2015 13:50:12 -0300
 Israel Meireles Chrisostomo israelmchrisost...@gmail.com escreveu:

  Alguém sabe se é possível provar que:seja k um natural,então se r^k é
  irracional então (r+1)^k também é irracional?
 

 --
 Esta mensagem foi verificada pelo sistema de antivírus e
  acredita-se estar livre de perigo.


 =
 Instru�ões para entrar na lista, sair da lista e usar a lista em
 http://www.mat.puc-rio.br/~obmlistas/obm-l.html
 =




-- 
Esdras Muniz Mota
Mestrando em Matemática
Universidade Federal do Ceará

-- 
Esta mensagem foi verificada pelo sistema de antiv�rus e
 acredita-se estar livre de perigo.



[obm-l] Re: [obm-l] Valor mínimo da função.

2015-05-04 Por tôpico Esdras Muniz
se os ais estão em ordem crescente, o mínimo é atingido no meio:
k= parte inteira de n/2
l=teto de n/2
se k=l, o mínimo é atingido em k, se kl o mínimo é atingido em qualquer
ponto de [k,l]

Em 4 de maio de 2015 10:55, Douglas Oliveira de Lima 
profdouglaso.del...@gmail.com escreveu:

 Olá caros amigos da lista, estou precisando confirmar um resultado que diz
 que o valor mínimo da expressão lx-a1l+lx-a2l+lx-a3l+...+lx-anl é assumido
 quando x=(a1+a2+a3+...+an)/n.
 Afina de contas qual o valor mínimo desta expressão? e para qual valor de
 x?

 Obrigado pela ajuda
 Abraços
 Douglas Oliveira.

 --
 Esta mensagem foi verificada pelo sistema de antivírus e
 acredita-se estar livre de perigo.




-- 
Esdras Muniz Mota
Mestrando em Matemática
Universidade Federal do Ceará

-- 
Esta mensagem foi verificada pelo sistema de antiv�rus e
 acredita-se estar livre de perigo.



[obm-l] Re: [obm-l] Valor mínimo da função.

2015-05-04 Por tôpico Esdras Muniz
Para ver, faça o caso n=2.

Em 4 de maio de 2015 11:23, Esdras Muniz esdrasmunizm...@gmail.com
escreveu:

 se os ais estão em ordem crescente, o mínimo é atingido no meio:
 k= parte inteira de n/2
 l=teto de n/2
 se k=l, o mínimo é atingido em k, se kl o mínimo é atingido em qualquer
 ponto de [k,l]

 Em 4 de maio de 2015 10:55, Douglas Oliveira de Lima 
 profdouglaso.del...@gmail.com escreveu:

 Olá caros amigos da lista, estou precisando confirmar um resultado que
 diz que o valor mínimo da expressão lx-a1l+lx-a2l+lx-a3l+...+lx-anl é
 assumido quando x=(a1+a2+a3+...+an)/n.
 Afina de contas qual o valor mínimo desta expressão? e para qual valor de
 x?

 Obrigado pela ajuda
 Abraços
 Douglas Oliveira.

 --
 Esta mensagem foi verificada pelo sistema de antivírus e
 acredita-se estar livre de perigo.




 --
 Esdras Muniz Mota
 Mestrando em Matemática
 Universidade Federal do Ceará





-- 
Esdras Muniz Mota
Mestrando em Matemática
Universidade Federal do Ceará

-- 
Esta mensagem foi verificada pelo sistema de antiv�rus e
 acredita-se estar livre de perigo.



[obm-l] Re: [obm-l] Re: [obm-l] Valor mínimo da função.

2015-05-04 Por tôpico Esdras Muniz
Isso mostra q o mínimo não é atingido na media.

Em 4 de maio de 2015 11:53, Esdras Muniz esdrasmunizm...@gmail.com
escreveu:

 Ponha por exemplo a1=0. a2=11, a3=12, a4=13 então, se f(x) =|x-0| +
 |x-11| +|x-12| +|x-13| , f(9)=9+2+3+4=18.
 enquanto f(11)= 11+0+1+2=14.

 Em 4 de maio de 2015 11:27, Pedro José petroc...@gmail.com escreveu:

 Bom dia!

 lx-a1l+lx-a2l+lx-a3l+...+lx-anl é mínimo ==

 f(x) =(x-a1)^2 + (x-a2)^2 + (x-a3)^2 + ...(x-an)^2 é mínimo.

 df/dx = 2nx - 2(a1+a2+a3+...+an) e d2f/dx^2 = 2n 0

 Logo se é mínimo == df/dx = 0 == 2nx = 2(a1+a2+a3+...+an) == x =
 (a1+a2+a3+...+an)/n

 Saudações,
 PJMS


 Em 4 de maio de 2015 11:19, Pedro José petroc...@gmail.com escreveu:

 Perdão, não havia entendido o enunciado.

 Saudações,
 PJMS

 Em 4 de maio de 2015 11:13, Pedro José petroc...@gmail.com escreveu:

 Bom dia!

 Como não há restrições para ai, 1= i = n., o mínimo valor é zero e
 ocorre quando x= ai = 0 para todo i, 1= i = n
 Um somatório de parcelas em módulo é =0 se ele atinge o valor zero é o
 mínimo.

 Se houver restriçoes para os ai, aí já muda de figura.

 Saudações,
 PJMS

 Em 4 de maio de 2015 10:55, Douglas Oliveira de Lima 
 profdouglaso.del...@gmail.com escreveu:

 Olá caros amigos da lista, estou precisando confirmar um resultado que
 diz que o valor mínimo da expressão lx-a1l+lx-a2l+lx-a3l+...+lx-anl é
 assumido quando x=(a1+a2+a3+...+an)/n.
 Afina de contas qual o valor mínimo desta expressão? e para qual valor
 de x?

 Obrigado pela ajuda
 Abraços
 Douglas Oliveira.

 --
 Esta mensagem foi verificada pelo sistema de antivírus e
 acredita-se estar livre de perigo.





 --
 Esta mensagem foi verificada pelo sistema de antivírus e
 acredita-se estar livre de perigo.




 --
 Esdras Muniz Mota
 Mestrando em Matemática
 Universidade Federal do Ceará





-- 
Esdras Muniz Mota
Mestrando em Matemática
Universidade Federal do Ceará

-- 
Esta mensagem foi verificada pelo sistema de antiv�rus e
 acredita-se estar livre de perigo.



[obm-l] Re: [obm-l] Re: [obm-l] Valor mínimo da função.

2015-05-04 Por tôpico Esdras Muniz
Ponha por exemplo a1=0. a2=11, a3=12, a4=13 então, se f(x) =|x-0| + |x-11|
+|x-12| +|x-13| , f(9)=9+2+3+4=18.
enquanto f(11)= 11+0+1+2=14.

Em 4 de maio de 2015 11:27, Pedro José petroc...@gmail.com escreveu:

 Bom dia!

 lx-a1l+lx-a2l+lx-a3l+...+lx-anl é mínimo ==

 f(x) =(x-a1)^2 + (x-a2)^2 + (x-a3)^2 + ...(x-an)^2 é mínimo.

 df/dx = 2nx - 2(a1+a2+a3+...+an) e d2f/dx^2 = 2n 0

 Logo se é mínimo == df/dx = 0 == 2nx = 2(a1+a2+a3+...+an) == x =
 (a1+a2+a3+...+an)/n

 Saudações,
 PJMS


 Em 4 de maio de 2015 11:19, Pedro José petroc...@gmail.com escreveu:

 Perdão, não havia entendido o enunciado.

 Saudações,
 PJMS

 Em 4 de maio de 2015 11:13, Pedro José petroc...@gmail.com escreveu:

 Bom dia!

 Como não há restrições para ai, 1= i = n., o mínimo valor é zero e
 ocorre quando x= ai = 0 para todo i, 1= i = n
 Um somatório de parcelas em módulo é =0 se ele atinge o valor zero é o
 mínimo.

 Se houver restriçoes para os ai, aí já muda de figura.

 Saudações,
 PJMS

 Em 4 de maio de 2015 10:55, Douglas Oliveira de Lima 
 profdouglaso.del...@gmail.com escreveu:

 Olá caros amigos da lista, estou precisando confirmar um resultado que
 diz que o valor mínimo da expressão lx-a1l+lx-a2l+lx-a3l+...+lx-anl é
 assumido quando x=(a1+a2+a3+...+an)/n.
 Afina de contas qual o valor mínimo desta expressão? e para qual valor
 de x?

 Obrigado pela ajuda
 Abraços
 Douglas Oliveira.

 --
 Esta mensagem foi verificada pelo sistema de antivírus e
 acredita-se estar livre de perigo.





 --
 Esta mensagem foi verificada pelo sistema de antivírus e
 acredita-se estar livre de perigo.




-- 
Esdras Muniz Mota
Mestrando em Matemática
Universidade Federal do Ceará

-- 
Esta mensagem foi verificada pelo sistema de antiv�rus e
 acredita-se estar livre de perigo.



Re: [obm-l] Elipse e areas

2015-05-08 Por tôpico Esdras Muniz
Sim.

Em 8 de maio de 2015 11:00, Douglas Oliveira de Lima 
profdouglaso.del...@gmail.com escreveu:

 Opa,  existe uma prova matemática para a teoria da segunda lei de Kepler?

 Douglas oliveira

 --
 Esta mensagem foi verificada pelo sistema de antivírus e
 acredita-se estar livre de perigo.




-- 
Esdras Muniz Mota
Mestrando em Matemática
Universidade Federal do Ceará

-- 
Esta mensagem foi verificada pelo sistema de antiv�rus e
 acredita-se estar livre de perigo.



[obm-l] Re: [Bulk] [obm-l] Princípio da casa dos pombos

2015-05-14 Por tôpico Esdras Muniz
fazendo para um conj com 4k elementos:
Defina A={x que pertence a (1, 2, ..., 4k) | x é par ou x2k+1},
desta forma temos k números que não estão em A, então vamos ter
obrigatoriamente que escolher pelo menos 2k+1 números de A. Vamos agora
separar A em 2k casas:
{1,2}, {2,4}, {3,6}, {4,8}, {5,10}, ..., {2k, 4k}.
Então pelo PCP, teremos que escolher dois números da mesma casa. Ou seja,
pegaremos dois números da forma x e 2x, dentre os 2k+1 que iremos escolher.

Em 14 de maio de 2015 22:01, Listeiro 037 listeiro_...@yahoo.com.br
escreveu:


 Eu concluí assim: para não ter múltiplo de um com o outro, eles devem
 ser primos. De 1 a 100 existem 25 números primos. Qualquer escolha acima
 de 25 no intervalo de 1 a 100 determina um novo número que é múltiplo
 de um dos 25 primeiros.

 Ou seja, existem 25 casas de pombos e 51 pombos.

 Em Thu, 14 May 2015 23:56:24 +
 marcone augusto araújo borges marconeborge...@hotmail.com escreveu:

  Do conjunto A = {1,2,...,99,100} escolhemos 51 números.Mostrar que
  entre os 51 números escolhidos,existem dois tais que um é múltiplo do
  outro.

 --
 Esta mensagem foi verificada pelo sistema de antivírus e
  acredita-se estar livre de perigo.


 =
 Instru�ões para entrar na lista, sair da lista e usar a lista em
 http://www.mat.puc-rio.br/~obmlistas/obm-l.html
 =




-- 
Esdras Muniz Mota
Mestrando em Matemática
Universidade Federal do Ceará

-- 
Esta mensagem foi verificada pelo sistema de antiv�rus e
 acredita-se estar livre de perigo.



Re: [obm-l] Resposta a Esdras(PCP)

2015-05-15 Por tôpico Esdras Muniz
Acho que o que eu escrevi ta errado. Mas pode ser feito assim:, vc pega
todos os números do conjunto (1, 2, ..., 2n) e escreve da forma (2^k)*I,
onde I é ímpar. Veja que o I vaira no conj dos ímpares de 1 a 2n-1, então
há n possibilidades para o I, então se vc pegar n+1 números no conj,
obrigatoriamente terá que pegar dois deles, x e y, com o mesmo I: x =
(2^a)*I e y = (2^b)*I, se ab, então x|y, se ab então y|x.

Em 15 de maio de 2015 08:38, marcone augusto araújo borges 
marconeborge...@hotmail.com escreveu:

 Na minha questão ficariam fora de A os números 51,53,...,97,99
 Escolheríamos os 25 números(k números) que estão fora de A
 Teríamos mais 25 pares de números de A: (1,2),(2,4),(3,6)...(50,100),
 dos quais teríamos obrigatoriamente que escolher 26 números dessas 25
 casas(no caso, k+1 números e não 2k+1)
 Então acabaríamos escolhendo dois números de uma mesma casa.
 Acho que nem conseguiria escolher 25 números, um de cada par, mas por
 isso, tudo bem.
 É isso?
 (Na minha ignorância ,a princípio, estranhei a repetição de números em
 casa distintas: 2 e 2,4 e 4,6 e 6...)
 Obrigado!!!



 --
 Esta mensagem foi verificada pelo sistema de antivírus e
 acredita-se estar livre de perigo.




-- 
Esdras Muniz Mota
Mestrando em Matemática
Universidade Federal do Ceará

-- 
Esta mensagem foi verificada pelo sistema de antiv�rus e
 acredita-se estar livre de perigo.



[obm-l] Re: [obm-l] Subconjuntos disjuntos e não vazios

2015-04-15 Por tôpico Esdras Muniz
(3¹⁰-2*2¹⁰+1)/2=28501
Vc escolhe um conj não vazio (10 escolhe k0), e multiplica pelo número de
formas de escolher um conjunto não vazio no complementar, soma com k
variando de 1 a 10, e divide por dois pois há repetência.

Em 15 de abril de 2015 17:28, Roger roger@gmail.com escreveu:

 Se alguém conseguir, agradeço.
 -

 Seja A o conjunto {1,2,3,4,5,6,7,8,9,10} . O número de conjuntos formados
 por dois subconjuntos disjuntos e não vazios de A é:

 a) 28500 b) 28501 c) 28502 d) 28503 e) 28504

 Att,
 Roger

 --
 Esta mensagem foi verificada pelo sistema de antivírus e
 acredita-se estar livre de perigo.




-- 
Esdras Muniz Mota
Mestrando em Matemática
Universidade Federal do Ceará

-- 
Esta mensagem foi verificada pelo sistema de antiv�rus e
 acredita-se estar livre de perigo.



[obm-l] Re: [obm-l] Soluções inteiras

2015-04-17 Por tôpico Esdras Muniz
É que os únicos restos possíveis de um quadrado por 11 são 1, 4, 9, 5 e 3.
Se houvesse solução inteira, o x² teria que ter resto 10 quando dividido
por 11.

Em 17 de abril de 2015 06:39, Pedro Chaves brped...@hotmail.com escreveu:

 Caros Colegas,

 Como podemos provar que a equação x^2 + 1 = 11y  não possui nenhuma
 solução inteira?

 Abraços!

 Pedro Chaves
 --
 Esta mensagem foi verificada pelo sistema de antivírus e
  acredita-se estar livre de perigo.


 =
 Instruções para entrar na lista, sair da lista e usar a lista em
 http://www.mat.puc-rio.br/~obmlistas/obm-l.html
 =




-- 
Esdras Muniz Mota
Mestrando em Matemática
Universidade Federal do Ceará

-- 
Esta mensagem foi verificada pelo sistema de antiv�rus e
 acredita-se estar livre de perigo.



[obm-l] Re: [obm-l] Triângulo russo 80-20-20

2015-04-09 Por tôpico Esdras Muniz
Tome P sobre AB de forma que o angulo PCB seja 70 graus. Prove que o
triangulo PCB e semelhante a CHD, caso lal.



Em quinta-feira, 9 de abril de 2015, Martins Rama martin...@pop.com.br
escreveu:

 O triângulo ABC é isósceles, com AB=AC e ângulos 20-80-80. Se H, que está
 sobre AB, é o pé da altura traçada a partir de C, e D é um ponto sobre AC
 tal que DC=BC/2, determine o ângulo CHD.
 Resp. 30.

 Olá pessoal.
 Vi hoje essa variação do triângulo russo 80-20-20, que ainda não resolvi.
 Alguma ideia?

 Abraços,
 Martins Rama.
 --
 Esta mensagem foi verificada pelo sistema de antivírus e
 acredita-se estar livre de perigo.



-- 
Esdras Muniz Mota
Mestrando em Matemática
Universidade Federal do Ceará

-- 
Esta mensagem foi verificada pelo sistema de antiv�rus e
 acredita-se estar livre de perigo.



Re: [obm-l] divisibilidade

2015-04-08 Por tôpico Esdras Muniz
999+1999000=11998999 =12x10⁶-1001=12x10⁶+3000-4000+1=(3000-1)(4000+1).

Em 8 de abril de 2015 12:04, Jefferson Franca jeffma...@yahoo.com.br
escreveu:

 Fiquei boa parte da madrugada tentando desvendar esse santo mistério:
 Mostre que o número 999+ 1999000 não é primo, ou seja, é composto.
 Será que alguém sabe como resolver esse problema interessante?
 Att
 Jefferson

 --
 Esta mensagem foi verificada pelo sistema de antivírus e
 acredita-se estar livre de perigo.




-- 
Esdras Muniz Mota
Mestrando em Matemática
Universidade Federal do Ceará

-- 
Esta mensagem foi verificada pelo sistema de antiv�rus e
 acredita-se estar livre de perigo.



[obm-l] Re: [obm-l] Dízima

2015-06-19 Por tôpico Esdras Muniz
Cara, acho q é alguma coisa do tipo (ordem de 10 na base 3^2005).Onde a
ordem de um número na base b é o menos natural k tal que a^k é congruente a
1 módulo b.

Em 19 de junho de 2015 11:05, Pedro Costa npc1...@gmail.com escreveu:

 Questão do livro( problemas selecionados de matemática - Gandbi- Pág.: 20
 questão : 63). Já faz dois anos que tento resolver
 este problema e não tem sucesso. Alguém de vocês poderia me ajudar.
 (questão: 63) Seja N o número de algarismos do período da dízima [image:
 \frac{1}{3^{2005}]. O número de algarismos de
 N é igual a:

 a) 952
 b) 953
 c) 954
 d) 955
 e) 956




 --
[image: Avast logo] http://www.avast.com/

 Este email foi escaneado pelo Avast antivírus.
 www.avast.com


 --
 Esta mensagem foi verificada pelo sistema de antivírus e
 acredita-se estar livre de perigo.




-- 
Esdras Muniz Mota
Mestrando em Matemática
Universidade Federal do Ceará

-- 
Esta mensagem foi verificada pelo sistema de antiv�rus e
 acredita-se estar livre de perigo.



[obm-l] Re: [obm-l] Dízima

2015-06-19 Por tôpico Esdras Muniz
Não é difícil de provar isso, daí vc usa o teorema de Euler pra calcular a
ordem: a^φ(n) é congruente a 1 módulo n se mdc(a,n)=1.

Em 19 de junho de 2015 11:55, Esdras Muniz esdrasmunizm...@gmail.com
escreveu:

 Cara, acho q é alguma coisa do tipo (ordem de 10 na base 3^2005).Onde a
 ordem de um número na base b é o menos natural k tal que a^k é congruente a
 1 módulo b.

 Em 19 de junho de 2015 11:05, Pedro Costa npc1...@gmail.com escreveu:

 Questão do livro( problemas selecionados de matemática - Gandbi- Pág.: 20
 questão : 63). Já faz dois anos que tento resolver
 este problema e não tem sucesso. Alguém de vocês poderia me ajudar.
 (questão: 63) Seja N o número de algarismos do período da dízima [image:
 \frac{1}{3^{2005}]. O número de algarismos de
 N é igual a:

 a) 952
 b) 953
 c) 954
 d) 955
 e) 956




 --
[image: Avast logo] http://www.avast.com/

 Este email foi escaneado pelo Avast antivírus.
 www.avast.com


 --
 Esta mensagem foi verificada pelo sistema de antivírus e
 acredita-se estar livre de perigo.




 --
 Esdras Muniz Mota
 Mestrando em Matemática
 Universidade Federal do Ceará





-- 
Esdras Muniz Mota
Mestrando em Matemática
Universidade Federal do Ceará

-- 
Esta mensagem foi verificada pelo sistema de antiv�rus e
 acredita-se estar livre de perigo.



[obm-l] Re: [obm-l] Re: [obm-l] Potência de primo

2015-05-19 Por tôpico Esdras Muniz
Se p|k então (p-1)|(p^(k-1) +p^(k-2)+...+1) pois p é congruente a 1 módulo
(p-1).
Mas nesse caso não pode ocorrer (p-1)!=p^k - 1 se k = p, pois podemos
mostrar por indução que
(n-1)!  n^n - 1 para todo natural maior que 1.

Em 18 de maio de 2015 20:34, Douglas Oliveira de Lima 
profdouglaso.del...@gmail.com escreveu:

 Considere que (p-1)!=p^k-1, com p5, e divida ambos os membros por p-1,
 assim teremos
 (p-2)!=p^(k-1) +p^(k-2)+...+1, o primeiro membro da equação possui um
 fator 2 e o fator (p-1)/2 então o primeiro membro possui um fator p-1, e o
 segundo membro da equação não possui este fator, assim não é possível a
 igualdade. E para p=1 o segundo membro da equação é igual a k diferente de
 zero.


 Douglas Oliveira

 Em 18 de maio de 2015 07:13, marcone augusto araújo borges 
 marconeborge...@hotmail.com escreveu:

 Seja p um número primo.Demonstrar que (p-1)! + 1 é uma potência de p se,
 e só se, p = 2, p= 3 ou p = 5.

 --
 Esta mensagem foi verificada pelo sistema de antivírus e
 acredita-se estar livre de perigo.



 --
 Esta mensagem foi verificada pelo sistema de antivírus e
 acredita-se estar livre de perigo.




-- 
Esdras Muniz Mota
Mestrando em Matemática
Universidade Federal do Ceará

-- 
Esta mensagem foi verificada pelo sistema de antiv�rus e
 acredita-se estar livre de perigo.



Re: [obm-l] livro

2015-06-26 Por tôpico Esdras Muniz
http://gen.lib.rus.ec/book/index.php?md5=44a6d2fcc779bdb0a35ef8ac769735e4

Em 25 de junho de 2015 19:24, Israel Meireles Chrisostomo 
israelmchrisost...@gmail.com escreveu:

 Olá alguém sabe onde posso encontrar um pdf para baixar do livro Wining
 Solutions de E. Lozansky. c. rousseau, se tiverem um link melhor ainda


 --
 Esta mensagem foi verificada pelo sistema de antivírus e
 acredita-se estar livre de perigo.




-- 
Esdras Muniz Mota
Mestrando em Matemática
Universidade Federal do Ceará

-- 
Esta mensagem foi verificada pelo sistema de antiv�rus e
 acredita-se estar livre de perigo.



Re: [obm-l] Sistema

2015-07-28 Por tôpico Esdras Muniz
Não, tome por exemplo a=b=c=2 e x=y=1 e z=4.

Em 28 de julho de 2015 00:27, Israel Meireles Chrisostomo 
israelmchrisost...@gmail.com escreveu:

 Se a+b+c=x+y+z então a²+b²+c²=x²+y²+z²?Isto é, uma coisa implica a outra?

 --
 Esta mensagem foi verificada pelo sistema de antivírus e
 acredita-se estar livre de perigo.




-- 
Esdras Muniz Mota
Mestrando em Matemática
Universidade Federal do Ceará

-- 
Esta mensagem foi verificada pelo sistema de antiv�rus e
 acredita-se estar livre de perigo.



[obm-l] Re: [obm-l] Equação

2015-08-11 Por tôpico Esdras Muniz
Olha o teorema 2.7 na pag 37 do livro de teoria dos números do José Plinio.

Em 11 de agosto de 2015 13:46, Israel Meireles Chrisostomo 
israelmchrisost...@gmail.com escreveu:

 x=5n+4 e y=3n+2 são as únicas soluções da equação 3x-5y=2?Em caso
 afirmativo, como provo que são as únicas soluções?

 --
 Esta mensagem foi verificada pelo sistema de antivírus e
 acredita-se estar livre de perigo.




-- 
Esdras Muniz Mota
Mestrando em Matemática
Universidade Federal do Ceará

-- 
Esta mensagem foi verificada pelo sistema de antiv�rus e
 acredita-se estar livre de perigo.



RE: [obm-l] Aritmética

2015-10-24 Por tôpico Esdras Muniz
Suponha um impar i>1, tal que i|mdc(a,b). Daí p=x^i+y^i=(x+y)() e tem-se um 
absurdo.

-Mensagem Original-
De: "marcone augusto araújo borges" 
Enviada em: ‎24/‎10/‎2015 23:58
Para: "obm-l@mat.puc-rio.br" 
Assunto: [obm-l] Aritmética

Alguém poderia resolver?


Sejam a, b, n, m inteiros positivos e suponha que a^n + b^m seja
um número primo.Mostre que (n,m) = 1 ou (n,m) = 2^r, para algum
r inteiro positivo.
Desde já agradeço.





-- 
Esta mensagem foi verificada pelo sistema de antivírus e 
acredita-se estar livre de perigo. 
-- 
Esta mensagem foi verificada pelo sistema de antiv�rus e
 acredita-se estar livre de perigo.



RE: [obm-l] Questão interessante

2015-11-03 Por tôpico Esdras Muniz
Se não há um dígito que aparece 3 vezes, então cada digito 0, 1, ..., 9 aparece 
duas vezes. Então a soma dos dígitos de p^n é 90, então 9|p^n.

-Mensagem Original-
De: "marcone augusto araújo borges" 
Enviada em: ‎03/‎11/‎2015 07:32
Para: "obm-l@mat.puc-rio.br" 
Assunto: [obm-l] Questão interessante

Seja p um número primo, p > 3.Sabe-se que para um certo inteiro positivo n
o número p^n possui 20 dígitos, quando escrito na base 10.Prove que dentre
esses dígitos existem pelo menos três iguais.


Eu tenho a solução.Estou compartilhando.

-- 
Esta mensagem foi verificada pelo sistema de antivírus e 
acredita-se estar livre de perigo. 
-- 
Esta mensagem foi verificada pelo sistema de antiv�rus e
 acredita-se estar livre de perigo.



RE: [obm-l] Matriz nxn

2015-11-03 Por tôpico Esdras Muniz
Dá (x-a)^{n-1}(x+(n-1)a). Eu fiz usando que esse determinante é um polinômio de 
grau n em x e coeficientes dependendo de a: "P^n(x,a)" (notação para o 
polinômio, de grau n,  do determinante desejado, em x e a). Daí temos que p(cx, 
ca)= c^nP(x, a). E, usando Chió, conseguimos: 
P^n(x,a)={(x-a)/x}^{n-1}P^{n-1}(x+a, a). O que implica que 
(x-a)^{n-1}|P^n(x,a). Agora, usando raízes (n-1)-esimas da unidade descobrimos 
que a outra raiz do polinômio é -(n-1)x.

Mas acho que conhecendo o resultado, deve ser mais fácil provar por indução...

-Mensagem Original-
De: "Anderson Torres" 
Enviada em: ‎03/‎11/‎2015 22:49
Para: "obm-l@mat.puc-rio.br" 
Assunto: Re: [obm-l] Matriz nxn

Você quer dizer algo assim, por exemplo?

X A A A A
A X A A A
A A X A A
A A A X A
A A A A X


Em 3 de novembro de 2015 23:42, Anderson Torres
 escreveu:
> Dê um exemplo. Não entendi nada.
>
> Em 3 de novembro de 2015 22:26, Eduardo Henrique
>  escreveu:
>> Pessoas, me deparei com a seguinte questão:
>>
>> Seja M uma matriz nxn com x na diagonal principal, e a>0 nas demais
>> posições. Calcule det(M).
>>
>> Alguém poderia me indicar um caminho para seguir? Eu não consegui avançar
>> nada nessa questão :(
>>
>> Att.
>>
>> Eduardo
>>
>> --
>> Esta mensagem foi verificada pelo sistema de antivírus e
>> acredita-se estar livre de perigo.

-- 
Esta mensagem foi verificada pelo sistema de antiv�rus e
 acredita-se estar livre de perigo.


=
Instru��es para entrar na lista, sair da lista e usar a lista em
http://www.mat.puc-rio.br/~obmlistas/obm-l.html
=

-- 
Esta mensagem foi verificada pelo sistema de antiv�rus e
 acredita-se estar livre de perigo.



RE: [obm-l] Problema 6 da OBM de 2002

2015-10-12 Por tôpico Esdras Muniz
Em qual EUREKA está a solução deste problema?


-Mensagem Original-
De: "Bernardo Freitas Paulo da Costa" 
Enviada em: ‎12/‎10/‎2015 12:29
Para: "Lista de E-mails da OBM" 
Assunto: Re: [obm-l] Problema 6 da OBM de 2002

2015-10-12 0:31 GMT-03:00 Gabriel Tostes :
> Mostre que não podemos formar mais que 4096 sequências binárias de tamanho 24 
> tal que quaisquer 2 diferem em ao menos 8 posições.
> Não consegui entender a resolução na Eureka. Alguém pode resolvê-lo?

Eu não sei se conheço alguma solução além da do Fábio (imagio que seja
esta a da Eureka). Mas acredito que pode ajudar a entender o problema
diminuindo os números, e tentando ser mais ambicioso: tente descobrir
a maior quantidade de sequências binárias de tamanho 4 tais que duas
quaisquer diferem em ao menos 2 posições. Vou tentar começar o
raciocínio: Suponha, sem perda de generalidade, que uma das sequências
é a . Então, você não pode ter nenhuma sequência com apenas um
"1", certo? Agora, pense em quantas sequências com dois "1" podem
haver. Ao todo, há 6, mas você não pode escolher todas elas, certo?

Para completar, ainda falta a idéia das "regiões de influência" (cada
sequência escolhida "domina" algumas sequências, as que estão mais
próximas dela do que de qualquer outra sequência). Para visualisar
isso, pense que, em vez de 4/2, o problema é sobre tam=5 / diferença
>= 3. Faça um "ponto" para cada sequência (dá 32, dá trabalho mas é
factível) e ligue as que diferem de apenas uma posição. Daí, comece
marcando uma (tipo a 0) e depois vá escolhendo como puder. Isso
deve deixar claro para você que cada sequência tem uma região de
influência de tamanho 1.

Abraços,
-- 
Bernardo Freitas Paulo da Costa

-- 
Esta mensagem foi verificada pelo sistema de antiv�rus e
 acredita-se estar livre de perigo.


=
Instru��es para entrar na lista, sair da lista e usar a lista em
http://www.mat.puc-rio.br/~obmlistas/obm-l.html
=

-- 
Esta mensagem foi verificada pelo sistema de antiv�rus e
 acredita-se estar livre de perigo.



[obm-l] Re: [obm-l] questão de treinamento olimpica.

2015-10-07 Por tôpico Esdras Muniz
Supondo por absurdo que isso ocorra, daí  temos que se a_i=11, então
b_i=11, do contrario, teríamos dois produtos de resto zero por 11. Então
vamos supor sem perda de generalidade que a_11=b_11=11.
daí, se x_i=a_i.b_i, supondo que {x_1,..., x_10} têm todos os restos
positivos possíveis por 11, então x_1.x_2...x_10 é congruente a (1.2...10)
(mod 11). (1.2...10)=(11-1)!.
Mas: x_1.x_2...x_10=(1.2...10)².
O teorema de Wilson garante que:
(11-1)! é congruente a -1 (mod 11). enquanto que (1.2...10)² é congruente a
1 (mod 11). O que é um absurdo.

Em 7 de outubro de 2015 08:59, Mauricio de Araujo <
mauricio.de.ara...@gmail.com> escreveu:

> Sejam a1, a2, ..., a11 e b1, b2, ..., b11 duas permutações dos inteiros 1,
> 2, ..., 11. Considere os números a1.b1, a2.b2, a3.b3, ..., a11.b11. Mostre
> que pelo menos dois destes números deixam o mesmo resto quando divididos
> por 11.
> Sugestão: Redução ao absurdo.
>
> --
> Abraços
>
> oɾnɐɹɐ ǝp oıɔıɹnɐɯ
>
>
> --
> Esta mensagem foi verificada pelo sistema de antivírus e
> acredita-se estar livre de perigo.




-- 
Esdras Muniz Mota
Mestrando em Matemática
Universidade Federal do Ceará

-- 
Esta mensagem foi verificada pelo sistema de antiv�rus e
 acredita-se estar livre de perigo.



RE: [obm-l] Desigualdade

2015-10-08 Por tôpico Esdras Muniz
Suponha spdg a>=d>=c. Daí, pela desigualdade do rearranjo, temos:
a(a^2/bc)+b(b^2/ac)+c(c^2/ab)>=(1/3)(a+b+c)(a^2/bc+b^2/ac+c^2/ab).
Daí vc usa MA>=MG pra mostrar que (a^2/bc+b^2/ac+c^2/ab)>=3. E acaba :)

-Mensagem Original-
De: "marcone augusto araújo borges" 
Enviada em: ‎08/‎10/‎2015 18:03
Para: "obm-l@mat.puc-rio.br" 
Assunto: [obm-l] Desigualdade

Sejam a, b e c números reais positivos.Mostre que
a^3/bc + b^3/ac + c^3/ab > = a + b + c

-- 
Esta mensagem foi verificada pelo sistema de antivírus e 
acredita-se estar livre de perigo. 
-- 
Esta mensagem foi verificada pelo sistema de antiv�rus e
 acredita-se estar livre de perigo.



RE: [obm-l] Contangentes

2015-09-15 Por tôpico Esdras Muniz
Verdade, vacilei, mas com o teorema de Chronecker dá pra mostrar que o conjunto 
cotg(Q) é denso em R.

-Mensagem Original-
De: "Israel Meireles Chrisostomo" <israelmchrisost...@gmail.com>
Enviada em: ‎15/‎09/‎2015 21:40
Para: "obm-l@mat.puc-rio.br" <obm-l@mat.puc-rio.br>
Assunto: Re: [obm-l] Contangentes

Além do mais, cos0=1 e sen0=0 a cotangente  não está definida entes ponto


Em 15 de setembro de 2015 21:33, Israel Meireles Chrisostomo 
<israelmchrisost...@gmail.com> escreveu:

Esqueci de dizer um valor que não seja tão trivial


Em 15 de setembro de 2015 21:25, Esdras Muniz <esdrasmunizm...@gmail.com> 
escreveu:

0.


Em terça-feira, 15 de setembro de 2015, Israel Meireles Chrisostomo 
<israelmchrisost...@gmail.com> escreveu:

Existe uma ângulo racional (em radianos) tal que a cotangente desse ângulo seja 
racional?

-- 
Esta mensagem foi verificada pelo sistema de antivírus e 
acredita-se estar livre de perigo. 


-- 

Esdras Muniz Mota
Mestrando em Matemática
Universidade Federal do Ceará





-- 
Esta mensagem foi verificada pelo sistema de antivírus e 
acredita-se estar livre de perigo. 





-- 
Esta mensagem foi verificada pelo sistema de antiv�rus e 
acredita-se estar livre de perigo. 
-- 
Esta mensagem foi verificada pelo sistema de antiv�rus e
 acredita-se estar livre de perigo.



Re: [obm-l] Contangentes

2015-09-15 Por tôpico Esdras Muniz
0.



Em terça-feira, 15 de setembro de 2015, Israel Meireles Chrisostomo <
israelmchrisost...@gmail.com> escreveu:

> Existe uma ângulo racional (em radianos) tal que a cotangente desse ângulo
> seja racional?
>
> --
> Esta mensagem foi verificada pelo sistema de antivírus e
> acredita-se estar livre de perigo.



-- 
Esdras Muniz Mota
Mestrando em Matemática
Universidade Federal do Ceará

-- 
Esta mensagem foi verificada pelo sistema de antiv�rus e
 acredita-se estar livre de perigo.



RE: [obm-l] Contangentes

2015-09-15 Por tôpico Esdras Muniz
Pois é, não implica em nada, so falei por falar.

-Mensagem Original-
De: "Israel Meireles Chrisostomo" <israelmchrisost...@gmail.com>
Enviada em: ‎15/‎09/‎2015 22:16
Para: "obm-l@mat.puc-rio.br" <obm-l@mat.puc-rio.br>
Assunto: Re: [obm-l] Contangentes

Isto implicaria em que?Que existem arcos racionais cujas cotangentes sejam 
racionais?Eu estou precisando de uma coisa do tipo, cara!


Em 15 de setembro de 2015 22:04, Esdras Muniz <esdrasmunizm...@gmail.com> 
escreveu:

Verdade, vacilei, mas com o teorema de Chronecker dá pra mostrar que o conjunto 
cotg(Q) é denso em R.


De: Israel Meireles Chrisostomo
Enviada em: ‎15/‎09/‎2015 21:40
Para: obm-l@mat.puc-rio.br
Assunto: Re: [obm-l] Contangentes


Além do mais, cos0=1 e sen0=0 a cotangente  não está definida entes ponto


Em 15 de setembro de 2015 21:33, Israel Meireles Chrisostomo 
<israelmchrisost...@gmail.com> escreveu:

Esqueci de dizer um valor que não seja tão trivial


Em 15 de setembro de 2015 21:25, Esdras Muniz <esdrasmunizm...@gmail.com> 
escreveu:

0.


Em terça-feira, 15 de setembro de 2015, Israel Meireles Chrisostomo 
<israelmchrisost...@gmail.com> escreveu:

Existe uma ângulo racional (em radianos) tal que a cotangente desse ângulo seja 
racional?

-- 
Esta mensagem foi verificada pelo sistema de antivírus e 
acredita-se estar livre de perigo. 


-- 

Esdras Muniz Mota
Mestrando em Matemática
Universidade Federal do Ceará





-- 
Esta mensagem foi verificada pelo sistema de antivírus e 
acredita-se estar livre de perigo. 





-- 

Esta mensagem foi verificada pelo sistema de antiv�rus e 
acredita-se estar livre de perigo. 
-- 
Esta mensagem foi verificada pelo sistema de antivírus e 
acredita-se estar livre de perigo. 



-- 
Esta mensagem foi verificada pelo sistema de antiv�rus e 
acredita-se estar livre de perigo. 
-- 
Esta mensagem foi verificada pelo sistema de antiv�rus e
 acredita-se estar livre de perigo.



Re: [obm-l] Contangentes

2015-09-15 Por tôpico Esdras Muniz
Isso parece difícil de provar. É exatamente esse o resultado que tu quer
provar, isso te ajudaria a resolver algum problema?

Em terça-feira, 15 de setembro de 2015, Israel Meireles Chrisostomo <
israelmchrisost...@gmail.com> escreveu:

> Aliás nem precisa ser n>k
>
> Em 15 de setembro de 2015 22:27, Israel Meireles Chrisostomo <
> israelmchrisost...@gmail.com
> <javascript:_e(%7B%7D,'cvml','israelmchrisost...@gmail.com');>> escreveu:
>
>> Com n e k inteiros e n>k
>>
>>
>> Em 15 de setembro de 2015 22:27, Israel Meireles Chrisostomo <
>> israelmchrisost...@gmail.com
>> <javascript:_e(%7B%7D,'cvml','israelmchrisost...@gmail.com');>> escreveu:
>>
>>> Por exemplo, eu precisaria provar que existe uma cotangente cujo arco
>>> seja na forma (2k+1)/2n, e tal que a cotangente desse arco seja racional,
>>> isto é possível?
>>>
>>> Em 15 de setembro de 2015 22:11, Israel Meireles Chrisostomo <
>>> israelmchrisost...@gmail.com
>>> <javascript:_e(%7B%7D,'cvml','israelmchrisost...@gmail.com');>>
>>> escreveu:
>>>
>>>> Isto implicaria em que?Que existem arcos racionais cujas cotangentes
>>>> sejam racionais?Eu estou precisando de uma coisa do tipo, cara!
>>>>
>>>> Em 15 de setembro de 2015 22:04, Esdras Muniz <
>>>> esdrasmunizm...@gmail.com
>>>> <javascript:_e(%7B%7D,'cvml','esdrasmunizm...@gmail.com');>> escreveu:
>>>>
>>>>> Verdade, vacilei, mas com o teorema de Chronecker dá pra mostrar que o
>>>>> conjunto cotg(Q) é denso em R.
>>>>> --
>>>>> De: Israel Meireles Chrisostomo
>>>>> <javascript:_e(%7B%7D,'cvml','israelmchrisost...@gmail.com');>
>>>>> Enviada em: ‎15/‎09/‎2015 21:40
>>>>> Para: obm-l@mat.puc-rio.br
>>>>> <javascript:_e(%7B%7D,'cvml','obm-l@mat.puc-rio.br');>
>>>>> Assunto: Re: [obm-l] Contangentes
>>>>>
>>>>> Além do mais, cos0=1 e sen0=0 a cotangente  não está definida entes
>>>>> ponto
>>>>>
>>>>> Em 15 de setembro de 2015 21:33, Israel Meireles Chrisostomo <
>>>>> israelmchrisost...@gmail.com
>>>>> <javascript:_e(%7B%7D,'cvml','israelmchrisost...@gmail.com');>>
>>>>> escreveu:
>>>>>
>>>>>> Esqueci de dizer um valor que não seja tão trivial
>>>>>>
>>>>>> Em 15 de setembro de 2015 21:25, Esdras Muniz <
>>>>>> esdrasmunizm...@gmail.com
>>>>>> <javascript:_e(%7B%7D,'cvml','esdrasmunizm...@gmail.com');>>
>>>>>> escreveu:
>>>>>>
>>>>>>> 0.
>>>>>>>
>>>>>>>
>>>>>>>
>>>>>>> Em terça-feira, 15 de setembro de 2015, Israel Meireles Chrisostomo <
>>>>>>> israelmchrisost...@gmail.com
>>>>>>> <javascript:_e(%7B%7D,'cvml','israelmchrisost...@gmail.com');>>
>>>>>>> escreveu:
>>>>>>>
>>>>>>>> Existe uma ângulo racional (em radianos) tal que a cotangente desse
>>>>>>>> ângulo seja racional?
>>>>>>>>
>>>>>>>> --
>>>>>>>> Esta mensagem foi verificada pelo sistema de antivírus e
>>>>>>>> acredita-se estar livre de perigo.
>>>>>>>
>>>>>>>
>>>>>>>
>>>>>>> --
>>>>>>> Esdras Muniz Mota
>>>>>>> Mestrando em Matemática
>>>>>>> Universidade Federal do Ceará
>>>>>>>
>>>>>>>
>>>>>>>
>>>>>>>
>>>>>>> --
>>>>>>> Esta mensagem foi verificada pelo sistema de antivírus e
>>>>>>> acredita-se estar livre de perigo.
>>>>>>
>>>>>>
>>>>>>
>>>>>
>>>>> --
>>>>> Esta mensagem foi verificada pelo sistema de antiv�rus e
>>>>> acredita-se estar livre de perigo.
>>>>> --
>>>>> Esta mensagem foi verificada pelo sistema de antivírus e
>>>>> acredita-se estar livre de perigo.
>>>>>
>>>>
>>>>
>>>
>>
>
> --
> Esta mensagem foi verificada pelo sistema de antivírus e
> acredita-se estar livre de perigo.



-- 
Esdras Muniz Mota
Mestrando em Matemática
Universidade Federal do Ceará

-- 
Esta mensagem foi verificada pelo sistema de antiv�rus e
 acredita-se estar livre de perigo.



Re: [obm-l] Contangentes

2015-09-15 Por tôpico Esdras Muniz
Errei tb [-1, 1].

Em terça-feira, 15 de setembro de 2015, Esdras Muniz <
esdrasmunizm...@gmail.com> escreveu:

> É denso em [0,1]. Basta tu ver o angulo como 360(n(1/pi) - m). E usar que
> cos é funçao contínua.
>
> Em terça-feira, 15 de setembro de 2015, Esdras Muniz <
> esdrasmunizm...@gmail.com
> <javascript:_e(%7B%7D,'cvml','esdrasmunizm...@gmail.com');>> escreveu:
>
>> Isso parece difícil de provar. É exatamente esse o resultado que tu quer
>> provar, isso te ajudaria a resolver algum problema?
>>
>> Em terça-feira, 15 de setembro de 2015, Israel Meireles Chrisostomo <
>> israelmchrisost...@gmail.com> escreveu:
>>
>>> Aliás nem precisa ser n>k
>>>
>>> Em 15 de setembro de 2015 22:27, Israel Meireles Chrisostomo <
>>> israelmchrisost...@gmail.com> escreveu:
>>>
>>>> Com n e k inteiros e n>k
>>>>
>>>>
>>>> Em 15 de setembro de 2015 22:27, Israel Meireles Chrisostomo <
>>>> israelmchrisost...@gmail.com> escreveu:
>>>>
>>>>> Por exemplo, eu precisaria provar que existe uma cotangente cujo arco
>>>>> seja na forma (2k+1)/2n, e tal que a cotangente desse arco seja racional,
>>>>> isto é possível?
>>>>>
>>>>> Em 15 de setembro de 2015 22:11, Israel Meireles Chrisostomo <
>>>>> israelmchrisost...@gmail.com> escreveu:
>>>>>
>>>>>> Isto implicaria em que?Que existem arcos racionais cujas cotangentes
>>>>>> sejam racionais?Eu estou precisando de uma coisa do tipo, cara!
>>>>>>
>>>>>> Em 15 de setembro de 2015 22:04, Esdras Muniz <
>>>>>> esdrasmunizm...@gmail.com> escreveu:
>>>>>>
>>>>>>> Verdade, vacilei, mas com o teorema de Chronecker dá pra mostrar que
>>>>>>> o conjunto cotg(Q) é denso em R.
>>>>>>> --
>>>>>>> De: Israel Meireles Chrisostomo
>>>>>>> Enviada em: ‎15/‎09/‎2015 21:40
>>>>>>> Para: obm-l@mat.puc-rio.br
>>>>>>> Assunto: Re: [obm-l] Contangentes
>>>>>>>
>>>>>>> Além do mais, cos0=1 e sen0=0 a cotangente  não está definida entes
>>>>>>> ponto
>>>>>>>
>>>>>>> Em 15 de setembro de 2015 21:33, Israel Meireles Chrisostomo <
>>>>>>> israelmchrisost...@gmail.com> escreveu:
>>>>>>>
>>>>>>>> Esqueci de dizer um valor que não seja tão trivial
>>>>>>>>
>>>>>>>> Em 15 de setembro de 2015 21:25, Esdras Muniz <
>>>>>>>> esdrasmunizm...@gmail.com> escreveu:
>>>>>>>>
>>>>>>>>> 0.
>>>>>>>>>
>>>>>>>>>
>>>>>>>>>
>>>>>>>>> Em terça-feira, 15 de setembro de 2015, Israel Meireles
>>>>>>>>> Chrisostomo <israelmchrisost...@gmail.com> escreveu:
>>>>>>>>>
>>>>>>>>>> Existe uma ângulo racional (em radianos) tal que a cotangente
>>>>>>>>>> desse ângulo seja racional?
>>>>>>>>>>
>>>>>>>>>> --
>>>>>>>>>> Esta mensagem foi verificada pelo sistema de antivírus e
>>>>>>>>>> acredita-se estar livre de perigo.
>>>>>>>>>
>>>>>>>>>
>>>>>>>>>
>>>>>>>>> --
>>>>>>>>> Esdras Muniz Mota
>>>>>>>>> Mestrando em Matemática
>>>>>>>>> Universidade Federal do Ceará
>>>>>>>>>
>>>>>>>>>
>>>>>>>>>
>>>>>>>>>
>>>>>>>>> --
>>>>>>>>> Esta mensagem foi verificada pelo sistema de antivírus e
>>>>>>>>> acredita-se estar livre de perigo.
>>>>>>>>
>>>>>>>>
>>>>>>>>
>>>>>>>
>>>>>>> --
>>>>>>> Esta mensagem foi verificada pelo sistema de antiv�rus e
>>>>>>> acredita-se estar livre de perigo.
>>>>>>> --
>>>>>>> Esta mensagem foi verificada pelo sistema de antivírus e
>>>>>>> acredita-se estar livre de perigo.
>>>>>>>
>>>>>>
>>>>>>
>>>>>
>>>>
>>>
>>> --
>>> Esta mensagem foi verificada pelo sistema de antivírus e
>>> acredita-se estar livre de perigo.
>>
>>
>>
>> --
>> Esdras Muniz Mota
>> Mestrando em Matemática
>> Universidade Federal do Ceará
>>
>>
>>
>>
>
> --
> Esdras Muniz Mota
> Mestrando em Matemática
> Universidade Federal do Ceará
>
>
>
>

-- 
Esdras Muniz Mota
Mestrando em Matemática
Universidade Federal do Ceará

-- 
Esta mensagem foi verificada pelo sistema de antiv�rus e
 acredita-se estar livre de perigo.



Re: [obm-l] Contangentes

2015-09-15 Por tôpico Esdras Muniz
É denso em [0,1]. Basta tu ver o angulo como 360(n(1/pi) - m). E usar que
cos é funçao contínua.

Em terça-feira, 15 de setembro de 2015, Esdras Muniz <
esdrasmunizm...@gmail.com> escreveu:

> Isso parece difícil de provar. É exatamente esse o resultado que tu quer
> provar, isso te ajudaria a resolver algum problema?
>
> Em terça-feira, 15 de setembro de 2015, Israel Meireles Chrisostomo <
> israelmchrisost...@gmail.com
> <javascript:_e(%7B%7D,'cvml','israelmchrisost...@gmail.com');>> escreveu:
>
>> Aliás nem precisa ser n>k
>>
>> Em 15 de setembro de 2015 22:27, Israel Meireles Chrisostomo <
>> israelmchrisost...@gmail.com> escreveu:
>>
>>> Com n e k inteiros e n>k
>>>
>>>
>>> Em 15 de setembro de 2015 22:27, Israel Meireles Chrisostomo <
>>> israelmchrisost...@gmail.com> escreveu:
>>>
>>>> Por exemplo, eu precisaria provar que existe uma cotangente cujo arco
>>>> seja na forma (2k+1)/2n, e tal que a cotangente desse arco seja racional,
>>>> isto é possível?
>>>>
>>>> Em 15 de setembro de 2015 22:11, Israel Meireles Chrisostomo <
>>>> israelmchrisost...@gmail.com> escreveu:
>>>>
>>>>> Isto implicaria em que?Que existem arcos racionais cujas cotangentes
>>>>> sejam racionais?Eu estou precisando de uma coisa do tipo, cara!
>>>>>
>>>>> Em 15 de setembro de 2015 22:04, Esdras Muniz <
>>>>> esdrasmunizm...@gmail.com> escreveu:
>>>>>
>>>>>> Verdade, vacilei, mas com o teorema de Chronecker dá pra mostrar que
>>>>>> o conjunto cotg(Q) é denso em R.
>>>>>> --
>>>>>> De: Israel Meireles Chrisostomo
>>>>>> Enviada em: ‎15/‎09/‎2015 21:40
>>>>>> Para: obm-l@mat.puc-rio.br
>>>>>> Assunto: Re: [obm-l] Contangentes
>>>>>>
>>>>>> Além do mais, cos0=1 e sen0=0 a cotangente  não está definida entes
>>>>>> ponto
>>>>>>
>>>>>> Em 15 de setembro de 2015 21:33, Israel Meireles Chrisostomo <
>>>>>> israelmchrisost...@gmail.com> escreveu:
>>>>>>
>>>>>>> Esqueci de dizer um valor que não seja tão trivial
>>>>>>>
>>>>>>> Em 15 de setembro de 2015 21:25, Esdras Muniz <
>>>>>>> esdrasmunizm...@gmail.com> escreveu:
>>>>>>>
>>>>>>>> 0.
>>>>>>>>
>>>>>>>>
>>>>>>>>
>>>>>>>> Em terça-feira, 15 de setembro de 2015, Israel Meireles Chrisostomo
>>>>>>>> <israelmchrisost...@gmail.com> escreveu:
>>>>>>>>
>>>>>>>>> Existe uma ângulo racional (em radianos) tal que a cotangente
>>>>>>>>> desse ângulo seja racional?
>>>>>>>>>
>>>>>>>>> --
>>>>>>>>> Esta mensagem foi verificada pelo sistema de antivírus e
>>>>>>>>> acredita-se estar livre de perigo.
>>>>>>>>
>>>>>>>>
>>>>>>>>
>>>>>>>> --
>>>>>>>> Esdras Muniz Mota
>>>>>>>> Mestrando em Matemática
>>>>>>>> Universidade Federal do Ceará
>>>>>>>>
>>>>>>>>
>>>>>>>>
>>>>>>>>
>>>>>>>> --
>>>>>>>> Esta mensagem foi verificada pelo sistema de antivírus e
>>>>>>>> acredita-se estar livre de perigo.
>>>>>>>
>>>>>>>
>>>>>>>
>>>>>>
>>>>>> --
>>>>>> Esta mensagem foi verificada pelo sistema de antiv�rus e
>>>>>> acredita-se estar livre de perigo.
>>>>>> --
>>>>>> Esta mensagem foi verificada pelo sistema de antivírus e
>>>>>> acredita-se estar livre de perigo.
>>>>>>
>>>>>
>>>>>
>>>>
>>>
>>
>> --
>> Esta mensagem foi verificada pelo sistema de antivírus e
>> acredita-se estar livre de perigo.
>
>
>
> --
> Esdras Muniz Mota
> Mestrando em Matemática
> Universidade Federal do Ceará
>
>
>
>

-- 
Esdras Muniz Mota
Mestrando em Matemática
Universidade Federal do Ceará

-- 
Esta mensagem foi verificada pelo sistema de antiv�rus e
 acredita-se estar livre de perigo.



[obm-l] Re: [obm-l] Função Sobrejetiva

2015-09-17 Por tôpico Esdras Muniz
Cara, vc pode fazer isso, pega duas sequências x_n e y_n, com
lim f(x_n)=+infinito elim f(y_n)=-infinito, e lim(x_n)=+infinito e
lim(y_n)=-infinito.
Daí tu usa que f é contínua.
vc pode pegar x_n=2kpi+pi/2 e y_n=-2kpi-pi/2.

Em 17 de setembro de 2015 12:27, Jeferson Almir <jefersonram...@gmail.com>
escreveu:

> 1. Provar que a função f( x ) = (x^3)sen( x ) é Sobrejetiva.
>
> A ideia que penso e que peço ajuda é que todo x real pode ser representado
> da forma x = 2kpi + 2/pi isso é válido ??? Caso seja, o problema está
> resolvido!!!
>
>
> --
> Esta mensagem foi verificada pelo sistema de antivírus e
> acredita-se estar livre de perigo.




-- 
Esdras Muniz Mota
Mestrando em Matemática
Universidade Federal do Ceará

-- 
Esta mensagem foi verificada pelo sistema de antiv�rus e
 acredita-se estar livre de perigo.



Re: [obm-l] Polinomios

2015-09-27 Por tôpico Esdras Muniz
Sim.

Em domingo, 27 de setembro de 2015, Israel Meireles Chrisostomo <
israelmchrisost...@gmail.com> escreveu:

> O polinômio P é um polinomio qualquer nas variáveis a,b e c , isto é a,b,c
> são variáveis
>
> Em 27 de setembro de 2015 17:02, Kelvin Anjos <kelvinan...@gmail.com
> <javascript:_e(%7B%7D,'cvml','kelvinan...@gmail.com');>> escreveu:
>
>> Quem são a,b,c? E o polinômio P?
>>
>>
>> Em 27 de setembro de 2015 16:19, Israel Meireles Chrisostomo <
>> israelmchrisost...@gmail.com
>> <javascript:_e(%7B%7D,'cvml','israelmchrisost...@gmail.com');>> escreveu:
>>
>>> Se eu provar que (a-b) divide um polinômio P, e depois provar que (a-c)
>>> divide o polinômio P, e depois provar que (b-c) divide o polinomio P, então
>>> eu posso dizer que o produto
>>> (a-b)(a-c)(b-c) divide o polinômio P?
>>>
>>> --
>>> Esta mensagem foi verificada pelo sistema de antivírus e
>>> acredita-se estar livre de perigo.
>>
>>
>>
>> --
>> Esta mensagem foi verificada pelo sistema de antivírus e
>> acredita-se estar livre de perigo.
>
>
>
> --
> Esta mensagem foi verificada pelo sistema de antivírus e
> acredita-se estar livre de perigo.



-- 
Esdras Muniz Mota
Mestrando em Matemática
Universidade Federal do Ceará

-- 
Esta mensagem foi verificada pelo sistema de antiv�rus e
 acredita-se estar livre de perigo.



Re: [obm-l] Pergunta que gera debates

2015-09-24 Por tôpico Esdras Muniz
-6

Em 24 de setembro de 2015 09:45, Mauricio de Araujo <
mauricio.de.ara...@gmail.com> escreveu:

> Perguntinha que gera debates, rsss
>
> Qual o resultado de
>
> sqrt(-4).sqrt(-9)?
>
> 6 ou -6?
>
> ​
> ​
>
> ​Evaluate:
>
> ​
>
> Abraços
>
> oɾnɐɹɐ ǝp oıɔıɹnɐɯ
>
>
> --
> Esta mensagem foi verificada pelo sistema de antivírus e
> acredita-se estar livre de perigo.




-- 
Esdras Muniz Mota
Mestrando em Matemática
Universidade Federal do Ceará

-- 
Esta mensagem foi verificada pelo sistema de antiv�rus e
 acredita-se estar livre de perigo.



[obm-l] Re: [obm-l] Re: [obm-l] duas séries e um resultado

2016-01-11 Por tôpico Esdras Muniz
vc quer calcular limite quando n vai pro infinito de:

 \frac{ \sum_{k=0}^n \frac{1}{\sqrt{2k+1}} }{  \sum_{k=1}^n
\frac{1}{\sqrt{2k}} } + 1 =
\frac{ \sum_{k=1}^n \frac{1}{\sqrt{k}} }{  \sum_{k=1}^n \frac{1}{\sqrt{2k}}
} =
\sqrt{2}  \frac{ \sum_{k=1}^2n \frac{1}{\sqrt{k}} }{  \sum_{k=1}^n
\frac{1}{\sqrt{k}} } =
\sqrt{2}  [1 + \frac{ \sum_{k=n}^2n \frac{1}{\sqrt{k + n}} }{  \sum_{k=1}^n
\frac{1}{\sqrt{k}} } ] =
\sqrt{2} + \sqrt{2}  \frac{ \sum_{k=n}^2n \frac{1}{\sqrt{k + n}} }{
\sum_{k=1}^n \frac{1}{\sqrt{k}} }

Mas \frac{ \sum_{k=n}^2n \frac{1}{\sqrt{k + n}} }{  \sum_{k=1}^n
\frac{1}{\sqrt{k}} } vai pra zero com n, para ver basta usar que a média
dos quadrados é maior ou igual a média aritmética, assim:

\sum_{k=n}^2n \frac{1}{\sqrt{k}} \leq \frac{1}{n}\sqrt{\sum_{k=n}^2n
\frac{1}{k + n}} <

\frac{1}{n}\sqrt{\sum_{k=n}^2n \frac{1}{n}} = \frac{1}{n}.

Enquanto \sum_{k=1}^n \frac{1}{\sqrt{k}} > 1.


Re: [obm-l] Ajuda numa desigualdade.

2016-01-28 Por tôpico Esdras Muniz
L = ((1+1/(n+1))^(n+1))/(1+1/n)^n = ((1 - 1/(n+1)²)^n)((n+2)/(n+1))

Use que (1 - x)^n > 1 - nx, Para x \in (0, 1)

L > (1 - n/(n+1)²)((n+2)/(n+1)) = ((n²+n+1)/(n²+2n+1))((n+2)/(n+1))
= (n³+3n²+3n+2)/(n³+3n²+3n+1) > 1.



Esse último termo é maior que 1.

Em 28 de janeiro de 2016 09:41, Douglas Oliveira de Lima <
profdouglaso.del...@gmail.com> escreveu:

> Opa Marcelo, muito obrigado mesmo, eu estou procurando uma solução
> daquelas tipo
> desigualdades, onde efetuamos uma estratégia para chegar no resultado,
> tipo uma daquelas que tu encontra no livro de combinatória do MOrgado(o
> problema das apostas).
> Mas valeu, se conseguir uma dessas me manda novamente por favor.
> Abraço
> Douglas Oliveira
>
> Em 28 de janeiro de 2016 01:26, Marcelo Salhab Brogliato <
> msbro...@gmail.com> escreveu:
>
>> Oi, Douglas, tudo bem?
>>
>> Se provarmos que f(x) = (1 + 1/x)^x é estritamente crescente, então está
>> provada sua desigualdade.
>>
>> Uma maneira é fazer isso usando cálculo. Seja g(x) = ln(f(x)) = x ln(1 +
>> 1/x). Assim, se provarmos que g(x) é estritamente crescente, então f(x)
>> também será (exercício: prove essa afirmação).
>>
>> g'(x) = ln(1 + 1/x) + x * (-1/x^2) / (1 + 1/x)  = ln(1 + 1/x) - (1/x) /
>> (1 + 1/x) = ln(1 + 1/x) - 1/(1+x)
>>
>> Temos que mostrar que g'(x) > 0 para todo x.
>>
>> Sabemos que ln(x) < x - 1, para x != 1. Aplicando essa desigualdade em
>> 1/x, temos: ln(1/x) < 1/x - 1 => ln(x) > 1 - 1/x, para x != 1.
>>
>> Aplicando a desigualdade acima em 1+1/x, temos: ln(1+1/x) > 1 - 1/(1 +
>> 1/x) = (1/x) / (1 + 1/x) = 1/(1+x). Logo: ln(1+1/x) > 1/(1+x) => g'(x) > 0
>> para todo x (já que 1+1/x > 1).
>>
>> Abraços,
>> Salhab
>>
>> 2016-01-28 0:34 GMT-02:00 Douglas Oliveira de Lima <
>> profdouglaso.del...@gmail.com>:
>>
>>> Olá caros amigos, gostaria de uma ajuda na seguinte desigualdade
>>> (1+1/n)^n<(1+1/n+1)^(n+1), para n natural.
>>>
>>> Agradeço desde já.
>>>
>>>
>>
>


-- 
Esdras Muniz Mota
Mestrando em Matemática
Universidade Federal do Ceará


[obm-l] Re: [obm-l] Dúvida função

2016-01-30 Por tôpico Esdras Muniz
Teorema: Se f: R ---> Y é contínua e X é compacto, então f admite um máximo
e um mínimo em X.

Em 28 de janeiro de 2016 23:16, Israel Meireles Chrisostomo <
israelmchrisost...@gmail.com> escreveu:

> Olá pessoal eu gostaria de provar que uma função admite máximo sem
> calcular o máximo da função, isto é possível?
> Por exemplo, seja f(a,b,c) uma função, eu quero provar que a,b,c admite
> máximo sem calcular seu máximo, lembrando f(a,b,c) é uma função de 3
> variáveis, alguém por favor poderia me ajudar?
>



-- 
Esdras Muniz Mota
Mestrando em Matemática
Universidade Federal do Ceará


RE: [obm-l] Fatorial e números primos

2016-04-01 Por tôpico Esdras Muniz
https://pt.m.wikipedia.org/wiki/Fatorial

Veja "Fatoração prima de fatoriais".

-Mensagem Original-
De: "Pedro Chaves" 
Enviada em: ‎01/‎04/‎2016 18:22
Para: "obm-l@mat.puc-rio.br" 
Assunto: [obm-l] Fatorial e números primos

Caros Colegas,

Proponho o teorema abaixo.

Teorema:

---  Na decomposição em fatores primos positivos do inteiro n >3,  o fator 2 
aparece mais vezes do que qualquer outro fator.  ---

Agradeço-lhes a atenção.

Pedro Chaves
---






-- 
Esta mensagem foi verificada pelo sistema de antivírus e 
acredita-se estar livre de perigo. 
-- 
Esta mensagem foi verificada pelo sistema de antiv�rus e
 acredita-se estar livre de perigo.



[obm-l] Re: [obm-l] nome de um quadrilátero

2016-08-10 Por tôpico Esdras Muniz
Rombo.

Em 10 de agosto de 2016 17:43, Luís Lopes <qed_te...@hotmail.com> escreveu:

> Sauda,c~oes,
>
>
> Qual o nome em português para o
>
> quadrilátero chamado de kite em inglês ?
>
>
>
> https://en.wikipedia.org/wiki/Kite_(geometry)
>
>
> Abs,
>
> Luís
>
>
>
> --
> Esta mensagem foi verificada pelo sistema de antivírus e
> acredita-se estar livre de perigo.
>



-- 
Esdras Muniz Mota
Mestrando em Matemática
Universidade Federal do Ceará

-- 
Esta mensagem foi verificada pelo sistema de antiv�rus e
 acredita-se estar livre de perigo.



[obm-l] Re: [obm-l] Re: [obm-l] Re: [obm-l] nome de um quadrilátero

2016-08-10 Por tôpico Esdras Muniz
Agora que vi na wikipédia acredito que o nome seja mesmo deltoide ou pipa
(prefiro o segundo), mas quando ainda estava no ensino médio um professor
me falou que o nome disso era rombo e eu acreditei até hoje.

Em 10 de agosto de 2016 19:02, Bruno Visnadi <brunovisnadida...@gmail.com>
escreveu:

> De acordo com o próprio Wikipédia, o nome é 'Deltoide' ou 'Pipa'.
>
> Em 10 de agosto de 2016 18:43, Luís Lopes <qed_te...@hotmail.com>
> escreveu:
>
>> Sauda,c~oes, oi Esdras,
>>
>> Obrigado. Difícil imaginar isso pois rhombus
>>
>> em inglês parece ser losango.
>>
>>
>> https://pt.wikipedia.org/wiki/Losango
>>
>>
>> Não me lembro de ter visto esse nome rombo.
>>
>>
>> Os livros didáticos usam esse nome para kite ?
>>
>>
>> Luís
>>
>> --
>> *De:* owner-ob...@mat.puc-rio.br <owner-ob...@mat.puc-rio.br> em nome de
>> Esdras Muniz <esdrasmunizm...@gmail.com>
>> *Enviado:* quarta-feira, 10 de agosto de 2016 21:13:26
>> *Para:* obm-l@mat.puc-rio.br
>> *Assunto:* [obm-l] Re: [obm-l] nome de um quadrilátero
>>
>> Rombo.
>>
>> Em 10 de agosto de 2016 17:43, Luís Lopes <qed_te...@hotmail.com>
>> escreveu:
>>
>>> Sauda,c~oes,
>>>
>>>
>>> Qual o nome em português para o
>>>
>>> quadrilátero chamado de kite em inglês ?
>>>
>>>
>>>
>>> https://en.wikipedia.org/wiki/Kite_(geometry)
>>>
>>>
>>> Abs,
>>>
>>> Luís
>>>
>>>
>>>
>>> --
>>> Esta mensagem foi verificada pelo sistema de antivírus e
>>> acredita-se estar livre de perigo.
>>>
>>
>>
>>
>> --
>> Esdras Muniz Mota
>> Mestrando em Matemática
>> Universidade Federal do Ceará
>>
>>
>>
>> --
>> Esta mensagem foi verificada pelo sistema de antiv�rus e
>> acredita-se estar livre de perigo.
>>
>> --
>> Esta mensagem foi verificada pelo sistema de antivírus e
>> acredita-se estar livre de perigo.
>>
>
>
> --
> Esta mensagem foi verificada pelo sistema de antivírus e
> acredita-se estar livre de perigo.
>



-- 
Esdras Muniz Mota
Mestrando em Matemática
Universidade Federal do Ceará

-- 
Esta mensagem foi verificada pelo sistema de antiv�rus e
 acredita-se estar livre de perigo.



[obm-l] Re: [obm-l] Qual a maior potência?

2017-01-16 Por tôpico Esdras Muniz
4^53 = 2^106 > 2^105 = (2^7)^15 = (128)^15 > 125^15 = 5^45 > 5^44.

Em 16 de janeiro de 2017 13:14, Douglas Oliveira de Lima <
profdouglaso.del...@gmail.com> escreveu:

> Olá amigos , gostaria de uma ajuda pra um raciocínio diferente, por log eu
> já fiz.
>
> Qual a maior potência? 4^53 ou 5^44.
>
> --
> Esta mensagem foi verificada pelo sistema de antivírus e
> acredita-se estar livre de perigo.




-- 
Esdras Muniz Mota
Mestrando em Matemática
Universidade Federal do Ceará

-- 
Esta mensagem foi verificada pelo sistema de antiv�rus e
 acredita-se estar livre de perigo.



[obm-l] Re: [obm-l] Re: Demosntração desigualdade

2016-09-08 Por tôpico Esdras Muniz
Dei uma olhada rapida, acho que as desigualdades 1, 2 e 3 nem sempre valem,
pois a concavidade da função tangente depende do intervalo em que o angulo
está. Mas o principal motivo da sua prova estar errada é vc achar que o k
vai poder "alcançar" o n, isso não pode acontecer pois vc está fazendo um
limite com o n indo para o infinito enquanto o k é fixo, pode ser muito
grande, mas é fixo.

Em 7 de setembro de 2016 13:24, Israel Meireles Chrisostomo <
israelmchrisost...@gmail.com> escreveu:

> Por favor alguém que entendeu o que fiz pode me ajudar a entender o que eu
> fiz de errado?
>
> Em 7 de setembro de 2016 12:37, Israel Meireles Chrisostomo <
> israelmchrisost...@gmail.com> escreveu:
>
>> Alguém pode me esclarecer o erro cometido na minha demonstração para esse
>> problema aqui:
>>
>> http://math.stackexchange.com/questions/1917400/inequality-o
>> n-six-variables
>>
>> O cara deu um contra exemplo que a desigualdade é falsa, mas não vejo
>> nenhum errro na minha demonstração, alguém poderia me dizer qual é o erro?
>>
>
>
> --
> Esta mensagem foi verificada pelo sistema de antivírus e
> acredita-se estar livre de perigo.
>



-- 
Esdras Muniz Mota
Mestrando em Matemática
Universidade Federal do Ceará

-- 
Esta mensagem foi verificada pelo sistema de antiv�rus e
 acredita-se estar livre de perigo.



[obm-l] Re: [obm-l] Teoria dos Números

2016-09-26 Por tôpico Esdras Muniz
Vou dar só uma dica: 3|(10^k)+2 para todo K natural.

Em 26 de setembro de 2016 16:37, Ricardo Leão <leaoricardo...@gmail.com>
escreveu:

> Seja n um inteiro não negativo. Prove que o número formado colocando 2^n e
> 2^(n+1) lado a lado em qualquer ordem é um múltiplo de 3.
>
> Eu tentei resolver usando congruência, mas eu travei nessa questão.
>
> Por favor, algum colega poderia fazer a demonstração?
>
> --
> Esta mensagem foi verificada pelo sistema de antivírus e
> acredita-se estar livre de perigo.




-- 
Esdras Muniz Mota
Mestrando em Matemática
Universidade Federal do Ceará

-- 
Esta mensagem foi verificada pelo sistema de antiv�rus e
 acredita-se estar livre de perigo.



RE: [obm-l] Ajuda em Aritmética

2016-09-26 Por tôpico Esdras Muniz
Se p é um primo diferente de 5, os restos dos outros 2 por 5 são os mesmos que 
os de p^2-1 e p^2+1 respectivamente. Se os 3 números são primos, nenhum deles é 
múltiplo de 5. Daí o produto (p^2-1)(p^2+1) não pode ser múltiplo de 5. Mas 
esse produto é p^4-1. Mas o pequeno teorema de Fermat garante que 5 divide 
p^4-1 se p for diferente de 4. Aí o problema acaba.

Se vc não quiser usar o pequeno teorema de Fermat, é só verificar que para r=1, 
2, 3 e 4, onde r é o resto de p por 5, ou 4p^2-1 ou 6p^2-1 é múltiplo de 5. 

Acho a primeira solução melhor pq mostra de onde o autor tirou a idéia de fazer 
a questão.

-Mensagem Original-
De: "Marcelo de Moura Costa" 
Enviada em: ‎26/‎09/‎2016 06:19
Para: "obm-l@mat.puc-rio.br" 
Assunto: [obm-l] Ajuda em Aritmética

Bom dia a todos, um anulo me apresentou esse problema e confesso que pela dica 
não consegui interpretá-lo corretamente e fiquei muito curioso como o mesmo, 
será que alguém poderia me ajudar?

O problema é:

Mostre que somente para p=5, os números p, 4p^2+1 e 6p^2+1 serão primos. (Dica: 
analise os restos da divisão de p por 5) 


Agradeço a atenção.



-- 
Esta mensagem foi verificada pelo sistema de antiv�rus e 
acredita-se estar livre de perigo. 
-- 
Esta mensagem foi verificada pelo sistema de antiv�rus e
 acredita-se estar livre de perigo.



RE: [obm-l] Divisibilidade Simultânea

2016-10-17 Por tôpico Esdras Muniz
Sim, m = n =1.

-Mensagem Original-
De: "Richard Vilhena" 
Enviada em: ‎17/‎10/‎2016 20:41
Para: "obm-l@mat.puc-rio.br" 
Assunto: [obm-l] Divisibilidade Simultânea

Gostaria que uma ajuda. Obrigado!


É possível encontrar inteiros m > 0, n > 0, tal que (n + 1)|(m2 + 1) e 
simultaneamente (m + 1)|(n2 + 1) ?


-- 
Esta mensagem foi verificada pelo sistema de antiv�rus e 
acredita-se estar livre de perigo. 
-- 
Esta mensagem foi verificada pelo sistema de antiv�rus e
 acredita-se estar livre de perigo.



[obm-l] Re: [obm-l] Re: [obm-l] Re: [obm-l] Re: [obm-l] Re: [obm-l] Re: [obm-l] Re: [obm-l] Re: [obm-l] Polinômio irredutível em Z

2016-11-24 Por tôpico Esdras Muniz
6h149740p847418
>>>>>>>>>
>>>>>>>>>
>>>>>>> --
>>>>>>> Esta mensagem foi verificada pelo sistema de antiv?rus e
>>>>>>>  acredita-se estar livre de perigo.
>>>>>>>
>>>>>>>
>>>>>>> 
>>>>>>> =
>>>>>>> Instru?es para entrar na lista, sair da lista e usar a lista em
>>>>>>> http://www.mat.puc-rio.br/~obmlistas/obm-l.html
>>>>>>> 
>>>>>>> =
>>>>>>>
>>>>>>>
>>>>>>>
>>>>>>
>>>>>>
>>>>>> 
>>>>>> This message was sent using IMP, the Internet Messaging Program.
>>>>>>
>>>>>>
>>>>>>
>>>>>> --
>>>>>> Esta mensagem foi verificada pelo sistema de antivírus e
>>>>>> acredita-se estar livre de perigo.
>>>>>>
>>>>>>
>>>>>> 
>>>>>> =
>>>>>> Instruções para entrar na lista, sair da lista e usar a lista em
>>>>>> http://www.mat.puc-rio.br/~obmlistas/obm-l.html
>>>>>> 
>>>>>> =
>>>>>>
>>>>>
>>>>>
>>>>> --
>>>>> Esta mensagem foi verificada pelo sistema de antivírus e
>>>>> acredita-se estar livre de perigo.
>>>>
>>>>
>>>> --
>>>> Esta mensagem foi verificada pelo sistema de antivírus e
>>>> acredita-se estar livre de perigo.
>>>>
>>>
>>>
>>> --
>>> Esta mensagem foi verificada pelo sistema de antivírus e
>>> acredita-se estar livre de perigo.
>>>
>>
>>
>> --
>> Esta mensagem foi verificada pelo sistema de antivírus e
>> acredita-se estar livre de perigo.
>>
>
>
> --
> Esta mensagem foi verificada pelo sistema de antivírus e
> acredita-se estar livre de perigo.
>



-- 
Esdras Muniz Mota
Mestrando em Matemática
Universidade Federal do Ceará

-- 
Esta mensagem foi verificada pelo sistema de antiv�rus e
 acredita-se estar livre de perigo.



Re: [obm-l] Problema de geometria.

2016-11-02 Por tôpico Esdras Muniz
O que são essas "flechas"?

Em 2 de novembro de 2016 17:57, Douglas Oliveira de Lima <
profdouglaso.del...@gmail.com> escreveu:

> Olá amigos , preciso de uma ajuda na seguinte questão, na verdade a
> resolução porque já tentei muita coisa, já aprendi muita coisa com ela, mas
> mesmo assim não a resolvi.
>
> As três flechas dos três lados (cordas) de um triângulo ABC inscrito em
> uma circunferência de raio R
> valem 1, 2 e 3 calcular a área do triângulo.
>
>
>
> Qualquer ajuda será bem vinda. Obrigado.
>
> Att . Douglas Oliveira
>
> --
> Esta mensagem foi verificada pelo sistema de antivírus e
> acredita-se estar livre de perigo.




-- 
Esdras Muniz Mota
Mestrando em Matemática
Universidade Federal do Ceará

-- 
Esta mensagem foi verificada pelo sistema de antiv�rus e
 acredita-se estar livre de perigo.



[obm-l] Recorrência

2016-10-16 Por tôpico Esdras Muniz
Olá amigos, gostaria que me passassem eferências de livros ou artigos que
falem sobre recorrência. Dês de já obrigado.

-- 
Esdras Muniz Mota
Mestrando em Matemática
Universidade Federal do Ceará

-- 
Esta mensagem foi verificada pelo sistema de antiv�rus e
 acredita-se estar livre de perigo.



[obm-l] Re: [obm-l] Re: [obm-l] Re: [obm-l] Recorrência

2016-10-16 Por tôpico Esdras Muniz
Muito obrigado.

Em 16 de outubro de 2016 20:49, Rodrigo Renji <rodrigo.uff.m...@gmail.com>
escreveu:

> Olá pessoal : )
> Estou escrevendo um material, se quiserem dar uma olhada, os links deixo
> abaixo
>
> ►(9.14) equações de diferenças ( recorrências lineares) I
> https://dl.dropboxusercontent.com/u/21174119/compartilhar/
> equacoesdiferencas.pdf
> ►(9.15) equações de diferenças ( recorrências lineares) II
> https://dl.dropboxusercontent.com/u/21174119/compartilhar/
> equacoesdediferenas2.pdf
>
> Em 16 de outubro de 2016 20:51, Jeferson Almir <jefersonram...@gmail.com>
> escreveu:
>
>> Principles and Techniques in Combinatorics
>> ( Chen chuan-chong ) acredito ser intermediário pra Phoda
>> Aí desses pesados existe o Introduction to Combinatorics e o
>> Problems in Combinatorics and Graph Theory ambos do renomado IOAN TOMESCU
>>
>> Em domingo, 16 de outubro de 2016, Esdras Muniz <
>> esdrasmunizm...@gmail.com> escreveu:
>>
>>> Olá amigos, gostaria que me passassem eferências de livros ou artigos
>>> que falem sobre recorrência. Dês de já obrigado.
>>>
>>> --
>>> Esdras Muniz Mota
>>> Mestrando em Matemática
>>> Universidade Federal do Ceará
>>>
>>>
>>>
>>> --
>>> Esta mensagem foi verificada pelo sistema de antivírus e
>>> acredita-se estar livre de perigo.
>>
>>
>> --
>> Esta mensagem foi verificada pelo sistema de antivírus e
>> acredita-se estar livre de perigo.
>>
>
>
> --
> Esta mensagem foi verificada pelo sistema de antivírus e
> acredita-se estar livre de perigo.
>



-- 
Esdras Muniz Mota
Mestrando em Matemática
Universidade Federal do Ceará

-- 
Esta mensagem foi verificada pelo sistema de antiv�rus e
 acredita-se estar livre de perigo.



Re: [obm-l] a quem possa interessar

2017-08-10 Por tôpico Esdras Muniz
Caramba, é muita coisa! Obrigado.

Em 10 de agosto de 2017 09:27, Carlos Gomes <cgomes...@gmail.com> escreveu:

> Obrigado por compartilhar Murício...uma mina de ouro!
>
> abraço, Cgomes.
>
> Em 9 de agosto de 2017 22:36, Mauricio de Araujo <
> mauricio.de.ara...@gmail.com> escreveu:
>
>> https://drive.google.com/drive/folders/0B8qeUE5SqcPAWFVaM1N5anN3S2M
>>
>>
>>
>> --
>> Esta mensagem foi verificada pelo sistema de antivírus e
>> acredita-se estar livre de perigo.
>
>
>
> --
> Esta mensagem foi verificada pelo sistema de antivírus e
> acredita-se estar livre de perigo.
>



-- 
Esdras Muniz Mota
Mestrando em Matemática
Universidade Federal do Ceará

-- 
Esta mensagem foi verificada pelo sistema de antiv�rus e
 acredita-se estar livre de perigo.



[obm-l] Re: [obm-l] Re: [obm-l] Re: [obm-l] Sugestão de material para OBM

2017-07-04 Por tôpico Esdras Muniz
Cara, vc deve começar pegando um bom livro de cálculo, recomendo o Calculus
do Apostol, nele vc vai aprender também álgebra linear e geometria
analítica. São dois volumes, é material para passar um ano estudando.
Enquanto isso vc pode também ir estudando pelo livro da Putnam, que o
Carlos Gomes citou, ele vai te dar uma visão geral do que é preciso saber
pra enfrenar as provas da olimpíada universitária. Não desanime por não
estar em um curso de exatas, Fermat é um dos matemáticos mais famosos da
história, e era um matemático amador...

Em 4 de julho de 2017 15:48, Pedro José <petroc...@gmail.com> escreveu:

> Boa tarde!
>
> Desculpe-me, pela intromissão. Mas você, que não é da área de exatas, fica
> difícil enveredar de cara no nível universitário.
> Procure começar pelo nível médio. Os problemas já são cascas-grossas.
> Primeiro se erguer, depois andar e por fim correr, é o que costumo dizer a
> minha filha.
>
> Saudações,
> PJMS
>
>
> Em 4 de julho de 2017 12:56, Carlos Gomes <cgomes...@gmail.com> escreveu:
>
>> Olá Max...não seu o seu histórico anterior com as Olimpiadas...mas nunca
>> é tarde para começar...para o nivel universitário uma ótima referência é o
>> livro (PUTNAM BEYOND)  https://libgen.pw/download.php?id=10688 . Uma
>> outra boa dica é o proprio site da OBM onde você encontra a revista EUREKA
>> e também as provas anteriores.
>>
>> Um grande abraço, Cgomes.
>>
>> Em 4 de julho de 2017 10:23, Max Alexandre <maxmalexan...@gmail.com>
>> escreveu:
>>
>>> Olá, pesoal!
>>>
>>> Sou iniciante na área de olimpíadas. Faço faculdade na área da saúde,
>>> então não verei muito calculo ao longo do curso. Mas estou realmente
>>> instigado a resolver problemas de matemática. Já até me inscrevi na OBM e
>>> na OMERJ. Por isso, peço encarecidamente sugestões de materias voltados a
>>> olimpíadas de matemática de nível universitário visando uma preparação mais
>>> sólida pras competições.
>>>
>>> Desde já, agradeço.
>>>
>>> --
>>> Esta mensagem foi verificada pelo sistema de antivírus e
>>> acredita-se estar livre de perigo.
>>
>>
>>
>> --
>> Esta mensagem foi verificada pelo sistema de antivírus e
>> acredita-se estar livre de perigo.
>>
>
>
> --
> Esta mensagem foi verificada pelo sistema de antivírus e
> acredita-se estar livre de perigo.
>



-- 
Esdras Muniz Mota
Mestrando em Matemática
Universidade Federal do Ceará

-- 
Esta mensagem foi verificada pelo sistema de antiv�rus e
 acredita-se estar livre de perigo.



Re: [obm-l] boatos sobre elon lages lima

2017-05-23 Por tôpico Esdras Muniz
Morreu sim, já faz alguns dias. :(

Em 23 de maio de 2017 21:00, Israel Meireles Chrisostomo <
israelmchrisost...@gmail.com> escreveu:

> É verdade que o Elon morreu?Fiquei chocado com essa notícia, o pessoal
> aqui poderia confirmar a veracidade dessa notícia?
>
> --
> Esta mensagem foi verificada pelo sistema de antivírus e
> acredita-se estar livre de perigo.




-- 
Esdras Muniz Mota
Mestrando em Matemática
Universidade Federal do Ceará

-- 
Esta mensagem foi verificada pelo sistema de antiv�rus e
 acredita-se estar livre de perigo.



Re: [obm-l] Desigualdade

2017-05-27 Por tôpico Esdras Muniz
Solução muito boa.

Em 27 de mai de 2017 00:37, "Gabriel Tostes"  escreveu:

> Tira ln, esse produto vai ser:
> Sum{n>=1} ln(n+1)/(2^n) = M
>
> Bora escrever M de outro jeito:
>
> M= ln(2) + [ln(3)-ln(2)]/2 + [ln(4)-ln(3)]/2^2 + ...
>
> M= Sum{n>=1} (ln(n+1)-ln(n))/2^(n-1)
>
> Como ln(n+1)-ln(n)=ln(1+1/n)<1/n
>
> M=2} 1/n.2^(n-1) = L + ln(2)
>
> Para achar L considere:
> 1/(1-x)= 1+x^2+x^3+...
>
> Integrando essa expressao temos que -(1/x).ln(1-x)= 1+x/2+x^2/3+...
> Substituindo x=1/2 achamos que L=2ln(2)-1
> E entao
> M< 3ln(2)-1 < ln(3)
>
>  E o produto pedido inicialmente eh menor que 3
>
>
>
>
>
>
>
>
> Sent from my iPad
> > On May 26, 2017, at 9:47 PM, Douglas Oliveira de Lima <
> profdouglaso.del...@gmail.com> wrote:
> >
> > Como posso fazer essa daqui:
> >
> > [2^(1/2)].[3^(1/4)].[4^(1/8)].[5^(1/16)]...<3
> >
> > Grande abraço a todos
> >
> > DouglasOliveira
> >
> > --
> > Esta mensagem foi verificada pelo sistema de antivírus e
> > acredita-se estar livre de perigo.
>
> --
> Esta mensagem foi verificada pelo sistema de antivírus e
>  acredita-se estar livre de perigo.
>
>
> =
> Instruções para entrar na lista, sair da lista e usar a lista em
> http://www.mat.puc-rio.br/~obmlistas/obm-l.html
> =
>

-- 
Esta mensagem foi verificada pelo sistema de antiv�rus e
 acredita-se estar livre de perigo.



Re: [obm-l] Desigualdade

2017-05-28 Por tôpico Esdras Muniz
Se vc colocar a1 igual a 0, 1 ou 2 vai ver queisso não é verdade. Acho que
é verdade se |a1|>e.

Em 28 de mai de 2017 11:58, "Douglas Oliveira de Lima" <
profdouglaso.del...@gmail.com> escreveu:

Então amigos, eu tive uma idéia mas não estou conseguindo concluir, vamos
lá:

Montei uma sequência e fiz a_1=3, e assim
[2^(1/2)].[3^(1/4)].[4^(1/8)].[5^(1/16)]...<3
se, e somente se,
[3^(1/4)].[4^(1/8)].[5^(1/16)]...<(3^2)/2, portanto a_2=(3^2)/2, e assim
sucessivamente escrevi a sequência
a_n=(a_(n-1)^2)/n, e usei um "leminha" para resolver, que é (a_n)^2>n+1, e
assim sai fácil, só não consegui escrever
a prova desse lema.

Mas com ele sai bem facil, pois se  (a_n)^2>n+1, então (a_n)>(n+1)^(1/2),
logo  (a_(n-1)^2)/n>(n+1)^(1/2), ou seja,
a_(n-1)>n^(1/2)(n+1)^(1/4), .,
a_1>[2^(1/2)].[3^(1/4)].[4^(1/8)].[5^(1/16)]...,
e como a_1=3, está provado.

Peço a ajuda de vocÊs para provar o lema.

 Lema:
Considere a sequência a_n=(a_(n-1)^2)/n, onde a_1=3 então (a_n)^2>n+1.



Douglas Oliveira




Em 27 de maio de 2017 18:10, Esdras Muniz <esdrasmunizm...@gmail.com>
escreveu:

> Solução muito boa.
>
> Em 27 de mai de 2017 00:37, "Gabriel Tostes" <gtos...@icloud.com>
> escreveu:
>
>> Tira ln, esse produto vai ser:
>> Sum{n>=1} ln(n+1)/(2^n) = M
>>
>> Bora escrever M de outro jeito:
>>
>> M= ln(2) + [ln(3)-ln(2)]/2 + [ln(4)-ln(3)]/2^2 + ...
>>
>> M= Sum{n>=1} (ln(n+1)-ln(n))/2^(n-1)
>>
>> Como ln(n+1)-ln(n)=ln(1+1/n)<1/n
>>
>> M=2} 1/n.2^(n-1) = L + ln(2)
>>
>> Para achar L considere:
>> 1/(1-x)= 1+x^2+x^3+...
>>
>> Integrando essa expressao temos que -(1/x).ln(1-x)= 1+x/2+x^2/3+...
>> Substituindo x=1/2 achamos que L=2ln(2)-1
>> E entao
>> M< 3ln(2)-1 < ln(3)
>>
>>  E o produto pedido inicialmente eh menor que 3
>>
>>
>>
>>
>>
>>
>>
>>
>> Sent from my iPad
>> > On May 26, 2017, at 9:47 PM, Douglas Oliveira de Lima <
>> profdouglaso.del...@gmail.com> wrote:
>> >
>> > Como posso fazer essa daqui:
>> >
>> > [2^(1/2)].[3^(1/4)].[4^(1/8)].[5^(1/16)]...<3
>> >
>> > Grande abraço a todos
>> >
>> > DouglasOliveira
>> >
>> > --
>> > Esta mensagem foi verificada pelo sistema de antivírus e
>> > acredita-se estar livre de perigo.
>>
>> --
>> Esta mensagem foi verificada pelo sistema de antivírus e
>>  acredita-se estar livre de perigo.
>>
>>
>> =
>> Instruções para entrar na lista, sair da lista e usar a lista em
>> http://www.mat.puc-rio.br/~obmlistas/obm-l.html
>> =
>>
>
> --
> Esta mensagem foi verificada pelo sistema de antivírus e
> acredita-se estar livre de perigo.
>


-- 
Esta mensagem foi verificada pelo sistema de antivírus e
acredita-se estar livre de perigo.

-- 
Esta mensagem foi verificada pelo sistema de antiv�rus e
 acredita-se estar livre de perigo.



Re: [obm-l] Radicais

2017-06-04 Por tôpico Esdras Muniz
Comece com a identidade: $n^2\,=\,1+(n-1)\sqrt{(n+1)^2}$.

$3=\sqrt{3^2}=\sqrt{1+2\sqrt{4^2}}=\sqrt{1+2\sqrt{1+3\sqrt{5^2}}}=\cdots$

Em 4 de junho de 2017 15:40, Douglas Oliveira de Lima <
profdouglaso.del...@gmail.com> escreveu:

> Opa amigo, o radical do Indiano Ramanujam, baixe um arquivo do Carlos
> Victor , muito bom tem esse problema resolvido e vários outros.
> Segue o link http://cursos.ufrrj.br/posgraduacao/profmat/
> dissertacoes/dissertacoe/
>
> Um abraço
> Douglas Oliveira.
>
> Em 4 de jun de 2017 3:19 PM, "Pedro Júnior" <pedromatematic...@gmail.com>
> escreveu:
>
>> Olá pessoal, vocês poderiam me ajudar a solucionar o problema abaixo? Já
>> vi alguns bem parecidos, mas esse está me pegando...
>>
>> Raiz (1+2Raiz(1+3Raiz(1+4Raiz(1+...= ?
>>
>> Desde já agradeço
>>
>> --
>> Esta mensagem foi verificada pelo sistema de antivírus e
>> acredita-se estar livre de perigo.
>
>
> --
> Esta mensagem foi verificada pelo sistema de antivírus e
> acredita-se estar livre de perigo.
>



-- 
Esdras Muniz Mota
Mestrando em Matemática
Universidade Federal do Ceará

-- 
Esta mensagem foi verificada pelo sistema de antiv�rus e
 acredita-se estar livre de perigo.



Re: [obm-l] desigualdade

2017-05-07 Por tôpico Esdras Muniz
Se vc faz S(x,y,z)=x/(x+y) + y/ (y+z) + z/(z+x). S>x/(x+y+z) + y/ (x+y+z) +
z/(z+y+x)=1.

Por outro lado, se vc toma x=1; n=n e z= 1/n, fica:
S(n)=1/(n+1)+n/(n+1)+(1/n)/(1+1/n) e se vc faz n tender para o infinito,
S(n) tende para 1.

Em 7 de maio de 2017 23:58, Anderson Torres <torres.anderson...@gmail.com>
escreveu:

> x/(x+y) + y/ (y+z) + z/(z+x)
>
> 1/(1+y/x) + 1/ (1+z/y) + 1/(1+x/z)
>
> 1/(1+A) + 1/ (1+B) + 1/(1+C) com ABC=1
>
> talvez dê para prosseguir
>
>
>
> Em 2 de maio de 2017 14:21, Pedro José <petroc...@gmail.com> escreveu:
> > Se pelo menos dois números forem iguais é fácil mostrar que a soma dará
> 1,5
> > <= 2.
> >
> > Para x, y e z diferentes, vamos supor x < y <z, o que dará a maior soma
> é x
> > , y = x+1 e z= y+1 = x+2
> >
> > teremos: x/(2x+1) + y/(2y+1) + z/(2z-2)
> >
> > é fácil ver que x/(2x+1) < 0,5 e y/(2y+1) < 0,5 ==> x/(2x+1) + y/(2y+1 <1
> >
> > (2z-2)/z = 2 -2/z, como x,y,z >0 e x < y < z ==> z>=3 ==> (2z-2)/z > 1
> ==>
> > z/(2z-2) <1 ==> x/(2x+1) + y/(2y+1) + z/(2z-2) < 2
> >
> > x/(x+y) + y/ (y+z) + z/(z+x) <= x/(2x+1) + y/(2y+1) + z/(2z-2) ==>
> x/(x+y) +
> > y/ (y+z) + z/(z+x) < 2.
> >
> > O sinal de desigualdade deve estar invertido.
> >
> > Saudações,
> > PJMS
> >
> > Em 30 de abril de 2017 21:32, Gabriel Tostes <gtos...@icloud.com>
> escreveu:
> >>
> >> Nem vi a condição de q era positivo, de fato n vale.
> >>
> >> Sent from my iPad
> >>
> >> On Apr 30, 2017, at 3:53 PM, Douglas Oliveira de Lima
> >> <profdouglaso.del...@gmail.com> wrote:
> >>
> >> Observe quando x=2, y=3 e z=1 a desigualdade não funciona, logo não
> >> basta substituir x+y=a,Â
> >> x+z=b e y+z=c, na verdade acho que  funciona ao "contrário" x/(x+y) +
> >> y/ (y+z) + z/(z+x) <= 2.
> >> A não ser que seja outra questão como por exemplo:
> >> (x+y)/z +(x+z)/y +(y+z)/x >=6 o que daria certo.
> >>
> >> Grande abraço
> >>
> >> Douglas Oliveira.
> >>
> >> Em 30 de abril de 2017 10:46, marcone augusto araújo borges
> >> <marconeborge...@hotmail.com> escreveu:
> >>>
> >>> Se x, y, z são números positivos, prove que x/(x+y) + y/ (y+z) +
> >>> z/(z+x) > = 2
> >>>
> >>>
> >>> --
> >>> Esta mensagem foi verificada pelo sistema de antivírus e
> >>> acredita-se estar livre de perigo.
> >>
> >>
> >>
> >> --
> >> Esta mensagem foi verificada pelo sistema de antivírus e
> >> acredita-se estar livre de perigo.
> >>
> >>
> >> --
> >> Esta mensagem foi verificada pelo sistema de antivírus e
> >> acredita-se estar livre de perigo.
> >
> >
> >
> > --
> > Esta mensagem foi verificada pelo sistema de antivírus e
> > acredita-se estar livre de perigo.
>
> --
> Esta mensagem foi verificada pelo sistema de antivírus e
>  acredita-se estar livre de perigo.
>
>
> =
> Instru�ões para entrar na lista, sair da lista e usar a lista em
> http://www.mat.puc-rio.br/~obmlistas/obm-l.html
> =
>



-- 
Esdras Muniz Mota
Mestrando em Matemática
Universidade Federal do Ceará

-- 
Esta mensagem foi verificada pelo sistema de antiv�rus e
 acredita-se estar livre de perigo.



Re: [obm-l] Problema de grafos

2017-09-02 Por tôpico Esdras Muniz
Cada vértice pode ter como grau um número de 0 a n-1, porém o 0 e o n-1 não
podem ambos ser graus de vértices, pois se um tem grau n-1 então ele está
ligado a todos os outros vértices. Então há apenas n-1 possibilidades para
o grau de cada vértice. Pelo pcp há dois vértices com o mesmo grau.

Em 2 de set de 2017 12:34 PM, "Daniel Rocha" 
escreveu:

> Bom dia,
>
> Seja G um grafo com n vértices, n maior que 1. Suponha que G não possua
> loops nem mais de uma aresta unindo pares de vértices. Prove que G possui
> dois vértices de graus iguais.
>
> Obrigado,
> Daniel
> --
> Esta mensagem foi verificada pelo sistema de antivírus e
>  acredita-se estar livre de perigo.
>
>
> =
> Instruções para entrar na lista, sair da lista e usar a lista em
> http://www.mat.puc-rio.br/~obmlistas/obm-l.html
> =
>

-- 
Esta mensagem foi verificada pelo sistema de antiv�rus e
 acredita-se estar livre de perigo.



Re: [obm-l] Fibonacci teoria dos numeros

2017-08-31 Por tôpico Esdras Muniz
Usa que f_{(m,n)}=(f_m, f_n)
Onde (a,b)=mdc(a,b).

Em 31 de agosto de 2017 16:30, Douglas Oliveira de Lima <
profdouglaso.del...@gmail.com> escreveu:

> Olá, como posso mostrar que para algum inteiro e positivo n, existe um
> número de Fibonacci que é múltiplo de n?
>
> Douglas Oliveira.
>
> --
> Esta mensagem foi verificada pelo sistema de antivírus e
> acredita-se estar livre de perigo.




-- 
Esdras Muniz Mota
Mestrando em Matemática
Universidade Federal do Ceará

-- 
Esta mensagem foi verificada pelo sistema de antiv�rus e
 acredita-se estar livre de perigo.



[obm-l] Re: [obm-l] Dúvida combinatória

2017-11-14 Por tôpico Esdras Muniz
Obs:

$$S(n,\,k_1,\cdots ,k_n)=\frac{n!}{(k_1!\cdots k_n!)(1!)^{k_1}\cdots
(n!)^{k_n}}$$

-- 
Esta mensagem foi verificada pelo sistema de antiv�rus e
 acredita-se estar livre de perigo.



[obm-l] Re: [obm-l] Dúvida combinatória

2017-11-14 Por tôpico Esdras Muniz
Bem, imagine que vc tem [image: n] bolas iguais e quer distribuí-las em
caixas de tamanhos [image: k_1,\,k_2,\,\cdots,k_n], onde na caixa [image:
k_i] cabe [image: i] bolas, e você quer que no final cada caixa esteja
totalmente cheia ou vazia. Isso é equivalente ao problema que você propõe,
e a resposta é:

[image: S(n,\,k_1,k_2,\cdots,k_t)=\frac{n!}{(k_1!\cdots
k_t!)(1!)^{k_1}\cdots(n!)^{k_n}}]

.

Em 13 de novembro de 2017 23:30, Eduardo Henrique <dr.dhe...@outlook.com>
escreveu:

> Pessoal, estava estudando o seguinte tipo de problema:
>
> Quantas são as soluções inteiras positivas de a+b+c=r, com r inteiro
> positivo. Até aqui ok. A dúvida veio depois:
>
> Quantas são as solução inteiras positivas de 1a+2b+3c=r? E mais
> geralmente, de 1k_1+...+n_kn=r? Alguém sabe como abordar esse tipo de
> problema ou então saberia me indicar um material de estudos?
>
> Obrigado.
>
> Eduardo
>
> --
> Esta mensagem foi verificada pelo sistema de antivírus e
> acredita-se estar livre de perigo.
>



-- 
Esdras Muniz Mota
Mestrando em Matemática
Universidade Federal do Ceará

-- 
Esta mensagem foi verificada pelo sistema de antiv�rus e
 acredita-se estar livre de perigo.



[obm-l] Re: [obm-l] área de triângulo( compartilhando)

2018-05-13 Por tôpico Esdras Muniz
Se u é o ângulo entre os lados de comprimento a e b, temos:
S = a*b*sen(u)/2 = (a^2+b^2)/4.
Daí, pela condição de igualdade entre as médias geométrica e aritmética,
temos que
sen(u)=1 e a=b. Logo os ângulos do triângulo são 90°, 45°, 45°.

Em 13 de maio de 2018 23:52, marcone augusto araújo borges <
marconeborge...@hotmail.com> escreveu:

> As medidas de dois lados de um triângulo são a e b e sua área é igual a
> (a^2+b^2)/4
>
> Determine os ângulos do triângulo
>
> --
> Esta mensagem foi verificada pelo sistema de antivírus e
> acredita-se estar livre de perigo.
>



-- 
Esdras Muniz Mota
Mestrando em Matemática
Universidade Federal do Ceará

-- 
Esta mensagem foi verificada pelo sistema de antiv�rus e
 acredita-se estar livre de perigo.



[obm-l] Re: [obm-l] Re: [obm-l] Re: [obm-l] Fwd: não sei como fazer, tentei desigualdades de médias e não saiu

2018-05-13 Por tôpico Esdras Muniz
Existem 85 triplas (p, q, r) com p<q escreveu:

>
> Em dom, 13 de mai de 2018 às 20:12, Pacini Bores <pacini.bo...@globo.com>
> escreveu:
>
>> Desculpe-me esqueci d colocar um dado na questão na hora d escrever. Os
> números p, q é r são primos ímpares. Havia colocado apenas ímpares.
>
>> Oi Daniel,
>>
>> Estranho, pois p=999, q= 1001 e r =1; teremos  p+q+r=2001 , pqr+1=
>> 100= (1000)^2.
>>
>> Ou seja, k=1000 ?
>>
>> Pacini
>>
>> Em 13/05/2018 2:56, Daniel Quevedo escreveu:
>>
>>
>>
>> - Mensagem encaminhada -
>> De: Daniel Quevedo <daniel...@gmail.com>
>> Data: dom, 13 de mai de 2018 às 02:54
>> Assunto:
>> Para: ob...@mat-puc.rio.br <ob...@mat-puc.rio.br>
>>
>>
>> Sabendo que p, q e r são números impares distintos com p+q+r= 2001 e que
>> k é um inteiro positivo tal que pqr +1 =k^2, a soma dos algarismos do único
>> valor possível para k é igual a:
>> A) 20
>> B) 21
>> C) 22
>> D) 23
>> E) 24
>> --
>> Fiscal: Daniel Quevedo
>> --
>> Fiscal: Daniel Quevedo
>>
>> --
>> Esta mensagem foi verificada pelo sistema de antivrus e
>> acredita-se estar livre de perigo.
>>
>>
>>
>> --
>> Esta mensagem foi verificada pelo sistema de antivírus e
>> acredita-se estar livre de perigo.
>>
> --
> Fiscal: Daniel Quevedo
>
> --
> Esta mensagem foi verificada pelo sistema de antivírus e
> acredita-se estar livre de perigo.
>



-- 
Esdras Muniz Mota
Mestrando em Matemática
Universidade Federal do Ceará

-- 
Esta mensagem foi verificada pelo sistema de antiv�rus e
 acredita-se estar livre de perigo.



Re: [obm-l] Combinatoria - quantas sequencias de comprimento "n" , com "p" elementos

2018-02-27 Por tôpico Esdras Muniz
p^n-(p-1)^n


Em 27 de fev de 2018 09:57, "Claudio Buffara" 
escreveu:

> Isso é igual ao número de sobrejeções de um conjunto com n elementos num
> conjunto com p elementos.
>
> É igual a p!*S(n,p), onde S(n,p) é o número de Stirling do 2o tipo (=
> número de partições de um conjunto com n elementos em p subconjuntos não
> vazios)
>
> Veja aqui: http://nptel.ac.in/courses/04026/lecture9.pdf  (eles usam
> m ao invés de p).
>
> []s,
> Claudio.
>
>
> 2018-02-27 7:25 GMT-03:00 Rogerio Ponce da Silva :
>
>> Ola' pessoal !
>> Existem quantas sequencias (diferentes entre si) de comprimento "n" ,
>> empregando-se somente "p" elementos, pelo menos uma vez cada um deles?
>>
>> []'s
>> Rogerio Ponce
>>
>> --
>> Esta mensagem foi verificada pelo sistema de antivírus e
>>  acredita-se estar livre de perigo.
>>
>> =
>> Instruções para entrar na lista, sair da lista e usar a lista em
>> http://www.mat.puc-rio.br/~obmlistas/obm-l.html
>> =
>>
>
>
> --
> Esta mensagem foi verificada pelo sistema de antivírus e
> acredita-se estar livre de perigo.

-- 
Esta mensagem foi verificada pelo sistema de antiv�rus e
 acredita-se estar livre de perigo.



Re: [obm-l] geometria plana

2018-07-29 Por tôpico Esdras Muniz
Pra mim não é tão fácil ver 3, to enferrujado na geometria plana. Pra
justificar acho que uma boa forma de ver é dizer que o triângulo APQ é
congruente ao PBC. Para concluir, o caminho mais curto que eu vi foi usar
que o triângulo MNR é semelhante ao BAN, e a razão é 1/2.

Em 28 de julho de 2018 20:51, Claudio Buffara 
escreveu:

> Idéia que me ocorreu: todo triângulo é afim-equivalente a um triângulo
> equilátero.
> Mediante translações, as medianas de um triângulo equilátero de lado 1
> formam um triângulo equilátero cujos lados medem raiz(3)/2 e, portanto,
> cuja área é 3/4.
> Será que uma transformação afim preserva a razão entre as áreas do
> triângulo original e do triângulo “medianico”?
>
> Abs,
> Cláudio.
>
> Enviado do meu iPhone
>
> Em 28 de jul de 2018, à(s) 19:08, matematica10complicada <
> profdouglaso.del...@gmail.com> escreveu:
>
> Então,podemos fazer o seguinte:
>
> Considere um triângulo ABC, cujas medianas são AM, BN, CP, e baricentro
> G desta forma
>
> 1)Monte um paralelogramo BNQM de forma que MQ intercepte AC em R.
>
> 2)Como o baricentro divide em seis áreas iguais, temos que a área do
> triângulo AGN será 1/6.
>
> 3)É fácil ver que MQ=BN, e AQ=CP.
>
> 4) Desta forma a área procurada será a do triângulo AMQ que é o dobro
> da área do triângulo AMR=3/8.
>
> Portanto a resposta é 3/4.
>
>
> Douglas Oliveira.
> Grande Abraço.
>
> Em 28 de julho de 2018 16:32, marcone augusto araújo borges <
> marconeborge...@hotmail.com> escreveu:
>
>> Seja um triangulo ABC cuja area eh igual a 1. Determinar a area do
>> triangulo cujos  lados sao iguais às medianas do triangulo ABC
>>
>> --
>> Esta mensagem foi verificada pelo sistema de antivírus e
>> acredita-se estar livre de perigo.
>>
>
>
> --
> Esta mensagem foi verificada pelo sistema de antivírus e
> acredita-se estar livre de perigo.
>
>
> --
> Esta mensagem foi verificada pelo sistema de antivírus e
> acredita-se estar livre de perigo.
>



-- 
Esdras Muniz Mota
Mestrando em Matemática
Universidade Federal do Ceará

-- 
Esta mensagem foi verificada pelo sistema de antiv�rus e
 acredita-se estar livre de perigo.



[obm-l] Re: [obm-l] Polinômios ( RPM)

2018-09-20 Por tôpico Esdras Muniz
Suponha por absurdo que (7-Ri)>=0 para toda raiz Ri, i=1,...,100.
Daí, por Ma>=Mg, temos:
1>=\sqer[100]{(7-R1)(7-R2)...(7-R100)}>1 então 1>1, o que é um absurdo.

Em sex, 21 de set de 2018 às 01:05, Jeferson Almir 
escreveu:

> Este problema é de uma R.P.M que não sei qual o exemplar e peço ajuda.
>
> Seja P(x) um polinômio de grau 100 tal que P(x) = x^100 -600x^99 +
> 98x^98+97x^97 +... + a_1x + a_o tem 100 raizes reais e que P(7) > 1 .
> Mostre que existe pelo menos uma raiz maior que 7 .
>
> Desconfio muito de usar médias mas não estou conseguindo adequar para
> aplica-las .
>
> --
> Esta mensagem foi verificada pelo sistema de antivírus e
> acredita-se estar livre de perigo.



-- 
Esdras Muniz Mota
Mestrando em Matemática
Universidade Federal do Ceará

-- 
Esta mensagem foi verificada pelo sistema de antiv�rus e
 acredita-se estar livre de perigo.



  1   2   >